Тепер Ви можете повідомити нас про будь-яку помилку в тесті, натиснувши кнопку
Гарячі клавіші Відкрити/закрити список тестів: Space Гортати тести: ← → Перекласти тест: Shift
Понято
0.0%
1 / 200
Хлопчик 12-ти років скаржиться на періодичний головний біль, серцебиття, дратівливість, непереносимість поїздок у транспорті. Скарги турбують 2 місяці. За цей час тричі відмічались напади підйому артеріального тиску до 140/100 мм рт.ст., це супроводжувалось підйомом температури до 38oC, тахікардією, тремором, відчуттям страху. Закінчувався напад значним сечовипусканням. Який найбільш імовірний діагноз? A 12-year-old boy complains of periodic headache, palpitations, irritability, intolerance to travel in transport. Complaints have been bothering him for 2 months. During this time, there were three attacks of blood pressure pressure up to 140/100 mm Hg, it was accompanied by a rise in temperature up to 38oC, tachycardia, tremors, a feeling of fear. The attack ended with significant urination. What is the most likely diagnosis?

Вегетативна дисфункція Vegetative dysfunction

Вроджена вада серця Congenital heart defect

Епілепсія Epilepsy

Міокардит Myocarditis

Пухлина мозку Brain tumor

2 / 200
Хлопчику 6-ти років був введений донорський імуноглобулін у зв’язку з контактом по вірусному гепатиту в сім’ї. По графіку вакцинації на цей час була запланована ревакцинація проти кору. Дитина оглянута дільничим педіатром та визнана здоровою. З якої причини лікар-імунолог не дав дозволу на вакцинацію? A 6-year-old boy was injected with donor immunoglobulin in connection with contact with viral hepatitis in the family. According to the vaccination schedule, a revaccination against measles. The child was examined by the district pediatrician and found to be healthy. Why did the immunologist not give permission for vaccination?

Можливий розвиток вірусного гепатиту на фоні вакцинації проти кору Possible development of viral hepatitis against the background of vaccination against measles

Неефективність вакцинації на фоні введеного імуноглобуліну Ineffectiveness of vaccination against the background of injected immunoglobulin

Висока імовірність захворювання на кір після вакцинації High probability of getting measles after vaccination

Можлива анафілактична реакція на вакцину Possible anaphylactic reaction to the vaccine

- -

3 / 200
Хвора 45-ти років скаржиться на захриплість голосу, яка триває протягом 7-ми років після перенесеної операції на щитоподібній залозі. Об’єктивно: слизова оболонка гортані рожева, голосові складки світлі. Під час фонації рухається тільки ліва голосова складка, права голосова складка знаходиться у серединному положенні. Який найбільш імовірний діагноз? A 45-year-old patient complains of hoarseness of voice, which has lasted for 7 years after the operation on the thyroid gland. Objectively: the mucous membrane of the larynx is pink, vocal folds are light. During phonation, only the left vocal fold moves, the right vocal fold is in the middle position. What is the most likely diagnosis?

Стеноз гортані в стадії компенсації Laryngeal stenosis in the stage of compensation

Лівобічний парез гортані Left-sided laryngeal paresis

Стеноз гортані в стадії неповної компенсації Laryngeal stenosis in the stage of incomplete compensation

Новоутворення правої голосової складки New formation of the right vocal fold

Правобічний парез гортані Right-sided laryngeal paresis

4 / 200
Хвору 22-х років через 5 тижнів після переохолодження турбує підвищення температури тіла, слабкість, м’язовий біль, неможливість самостійно рухатись. Об’єктивно: болючість, ущільнення м’язів плечей, гомілок; активні рухи мінімальні; еритема грудної клітки спереду. Періорбітальний набряк з геліотропною еритемою. Позитивний симптом Готтрона. Яке дослідження необхідно провести для верифікації діагнозу? 5 weeks after hypothermia, a 22-year-old patient is concerned about increased body temperature, weakness, muscle pain, inability to move independently. Objectively: soreness, tightness muscles of the shoulders, lower legs; active movements are minimal; erythema of the chest in front. Periorbital edema with heliotropic erythema. Positive Gottron's symptom. What research should be done to verify the diagnosis?

Активність амінотрансфераз Activity of aminotransferases

Ревматоїдний фактор Rheumatoid factor

Біопсія м’язів Muscle biopsy

Титр АСЛО ASLO title

Рентгенографія суглобів X-ray of joints

5 / 200
Хвора 32-х років скаржиться на біль у епігастрії, що виникає вночі і зранку натще та через 2 години після прийому їжі, блювання 'кавовою гущею', печію, періодичні запаморочення, загальну слабкість. Об’єктивно: болючість в гастродуоденальній зоні, позитивний симптом Менделя. У крові: НЬ- 90 г/л; лейк.- 8,0-109 /л; ШЗЕ- 20 мм/год. Чим ускладнився перебіг захворювання? A 32-year-old patient complains of pain in the epigastrium that occurs at night and in the morning on an empty stomach and 2 hours after eating, vomiting of 'coffee grounds', heartburn, periodic dizziness, general weakness. Objectively: pain in the gastroduodenal zone, a positive Mendelian symptom. In the blood: Hb - 90 g/l; leuk. - 8.0-109 /l; SZE - 20 mm/h. What made the course more complicated disease?

Перфорація Perforation

Стеноз Stenosis

Малігнізація Malignancy

Пенетрація Penetration

Шлунково-кишкова кровотеча Gastrointestinal bleeding

6 / 200
Хворий 24-х років звернувся до лікаря зі скаргами на збільшення підщелепних лімфовузлів. Об’єктивно: збільшені підщелепні, пахвові та пахвинні лімфатичні вузли. На рентгенограмі органів грудної клітки - збільшені лімфовузли середостіння. У крові: ер.-3,4 • 1012/л, нЬ- 100 г/л, КП- 0,88, тр.-190- 109 /л, лейк.- 7,5- 109 /л, е,- 8%, п,- 2%, с.- 67%, лімф.- 23%, ШОЕ- 22 мм/год. Яке дослідження показане для верифікації причини лімфаденопатії? A 24-year-old patient turned to the doctor with complaints of enlarged submandibular lymph nodes. Objectively: enlarged submandibular, axillary and inguinal lymph nodes. On chest X-ray - enlarged lymph nodes of the mediastinum. In the blood: er.-3.4 • 1012/l, nb- 100 g/l, KP- 0.88, tr.-190- 109 /l, leuk.- 7.5- 109 / l, e, - 8%, p, - 2%, s. - 67%, lymph. - 23%, SOE - 22 mm/h. What research is indicated to verify the cause of lymphadenopathy?

Пункційна біопсія лімфатичних вузлів Puncture biopsy of lymph nodes

Стернальна пункція Sternal puncture

Відкрита біопсія лімфатичних вузлів Open biopsy of lymph nodes

Томографія середостіння Tomography of the mediastinum

Ультразвукове дослідження органів черевної порожнини Ultrasound examination of abdominal organs

7 / 200
У хворого 35-ти років на 8-му добу після хірургічної обробки гнійного осередку рана очистилась від гнійно-некротичного вмісту, з’явились грануляції. Проте, на фоні антибактеріальної терапії температура тіла тримається на рівні 38,5 — 39,5oC. Озноби, пітливість, ейфорія, Ps- 120/хв. Про яке ускладнення місцевого гнійно-запального процесу можна думати? In a 35-year-old patient, on the 8th day after surgical treatment of a purulent center, the wound was cleared of purulent-necrotic contents, granulations appeared. However, in the background after antibacterial therapy, the body temperature is kept at the level of 38.5 — 39.5oC. Chills, sweating, euphoria, Ps- 120/min. What complication of the local purulent-inflammatory process can you think about?

Запалення легень Pneumonia

Гнійно-резорбтивна лихоманка Suppurative resorptive fever

Сепсис Sepsis

Тромбофлебіт Thrombophlebitis

Менінгіт Meningitis

8 / 200
У фарбувальника автозаводу встановлено дiагноз гострого отруєння амідосполуками бензолу середнього ступеня важкості. Після стаціонарного лікування має місцє значне покращання стану. Яке експертне рішення слід прийняти в даному випадку? A painter at a car factory has been diagnosed with acute poisoning by benzene amido compounds of moderate severity. After inpatient treatment, the condition has improved significantly. What expert decision should be made in this case?

Видати листок непрацездатності для амбулаторного лікування Issue a certificate of incapacity for work for outpatient treatment

Направити на МСЕК для визначення відсотка втрати професійної працездатності Send to MSEK to determine the percentage of loss of professional capacity

Видати 'профбюлетень' для амбулаторного лікування Issue 'professional bulletin' for outpatient treatment

Може продовжувати роботу при дотриманні санітарно-гігієнічних норм Can continue to work subject to compliance with sanitary and hygienic standards

Направити на МСЕК для встановлення групи інвалідності в зв’язку з професійним Send to MSEK to establish a disability group in connection with professional

9 / 200
Хвора 20-ти років звернулася до лікаря жіночої консультації зі скаргами на свербіж, печіння і біль у ділянці зовнішніх статевих органів, підвищення температури тіла, дратівливість, порушення сну, головний біль. Хвора незаміжня, веде безладне статеве життя. Під час огляду зовнішніх статевих органів на фоні гіперемованої, набряклої слизової оболонки визначаються везикули, група яких займає до 2 см ураженої поверхні. Яка найбільш імовірна причина зумовлює таку клінічну картину? A 20-year-old female patient turned to a female doctor with complaints of itching, burning and pain in the area of the external genitalia, increased body temperature, irritability, sleep disturbances, headache. The patient is unmarried, leads a promiscuous sexual life. During the examination of the external genitalia, vesicles are identified against the background of a hyperemic, swollen mucous membrane, a group of which occupies up to 2 cm of the affected surface. What is the most likely reason for such a clinical picture?

Цитомегаловірусна інфекція Cytomegalovirus infection

Генітальна герпетична інфекція Genital herpes infection

Папіломавірусна інфекція Papillomavirus infection

Первинний сифіліс Primary syphilis

Кандиломатоз Condylomatosis

10 / 200
Хворому 51-го року проведена успішна серцево-легенева реанімація після суїцидальної спроби шляхом повішання. Через 1,5 години: непритомний, АТ-130/90 мм рт.ст., Ps130/хв. Наростає дихальна недостатність: ЧД- 28/хв., задишка інспіраторного характеру, втяжіння надключичних ямок на вдосі, у диханні беруть участь крила носа. Аускультативно: над трахеєю грубий свист, над легенями - жорстке дихання. Яка основна причина дихальної недостатності у хворого? A 51-year-old patient underwent successful cardiopulmonary resuscitation after a suicide attempt by hanging. 1.5 hours later: unconscious, blood pressure 130/90 mm Hg. st., Ps130/min. Respiratory insufficiency increases: BH - 28/min, dyspnea of an inspiratory nature, retracting the supraclavicular fossa on the left side, the wings of the nose take part in breathing. Auscultation: rough whistling over the trachea, rough breathing over the lungs. What is the main the cause of respiratory failure in the patient?

Аспіраційний синдром Aspiration syndrome

Ушкодження хрящової гортані Damage of cartilaginous larynx

Нейрогенний набряк легень Neurogenic pulmonary edema

Пригнічення дихального центру Suppression of the respiratory center

Бронхообтураційний синдром Broncho-obstructive syndrome

11 / 200
В жіночу консультацію звернулася жінка 40-ка років зі скаргами на порушення менструального циклу по типу гіперполіменореї протягом півроку, тягнучі болі в нижніх відділах живота, слабкість. При гінекологічному обстеженні тіло матки збільшене до 12 тижнів вагітності, щільне, рухоме, безболісне. В крові: Hb- 90 г/л. Яку патологію можна запідозрити? A 40-year-old woman came to the women's consultation with complaints of irregular menstrual cycles in the form of hyperpolymenorrhea for six months, pulling pains in the lower abdomen, weakness. During a gynecological examination the body of the uterus is enlarged up to 12 weeks of pregnancy, dense, mobile, painless. In the blood: Hb - 90 g/l. What pathology can be suspected?

Кістома яєчника Ovarian cystoma

Вагітність Pregnancy

Міома матки Uterine myoma

Дисфункціональна маткова кровотеча Dysfunctional uterine bleeding

Рак тіла матки Cancer of the uterine body

12 / 200
Хвора 25-ти років скаржиться на значне зниження зору на праве око, що виникло раптово. При обстеженні окуліста: ознаки ретробульбарного невриту. Об’єктивно: черевні рефлекси відсутні, вібраційна чутливість знижена. Встановіть діагноз: A 25-year-old patient complains of a significant decrease in vision in the right eye, which occurred suddenly. During the ophthalmologist's examination: signs of retrobulbar neuritis. Objectively: abdominal reflexes are absent , vibration sensitivity is reduced. Make a diagnosis:

Розсіяний склероз Multiple sclerosis

Бічний аміотрофічний склероз Amyotrophic lateral sclerosis

Пухлина головного мозку Brain tumor

Фунікулярний мієлоз Funicular myelosis

Атрофія зорових нервів Atrophy of optic nerves

13 / 200
Повторнонароджуюча 26-ти років прибула в пологове відділення у зв’язку з вагітністю 40 тижнів та початком пологової діяльності. 2 години тому відійшли навколоплідні води. Положення плоду повздовжнє, головне передлежання. ОЖ- 100 см, ВДМ- 42 см. Перейми через 4-5 хвилин, по 25 секунд. При внутрішньому акушерському обстеженні: шийка матки згладжена, відкриття 4 см. Плідного міхура немає. Головка плоду притиснута до входу в малий таз. Яке ускладнення виникло в пологах? A 26-year-old woman who gave birth again came to the maternity ward due to her 40-week pregnancy and the beginning of labor. 2 hours ago, the amniotic fluid broke. The position of the fetus is longitudinal, main presentation. OZ - 100 cm, VDM - 42 cm. Take after 4-5 minutes, 25 seconds each. During internal obstetric examination: the cervix is smooth, the opening is 4 cm. There is no fetal bladder. The head of the fetus is pressed against the entrance to the pelvis. What complication occurred during childbirth?

Вторинна слабкість пологової діяльності Secondary weakness of labor activity

Дискоординована пологова діяльність Discoordinated birth activity

Передчасне вилиття навколоплідних вод Premature discharge of amniotic fluid

Клінічно вузький таз Clinically narrow pelvis

Первинна слабкість пологової діяльності Primary weakness of labor activity

14 / 200
Хворий 38-ми років поступив у наркологічне відділення із скаргами на безсоння, тривогу. Ввечері став неспокійним, зривався з ліжка, струшував з ковдри і одягу 'павуків', скрізь бачив 'здорових щурів і змій', відганяв неіснуючих мух. Вважає, що він дома, чує за вікном 'голоси циган', хоче бігти, щоб їх 'вигнати з городу'. Соматичний стан: температура - 37,8oC, виражений гіпергідроз, тремор усього тіла. Який найбільш імовірний діагноз? A 38-year-old patient was admitted to the narcology department with complaints of insomnia and anxiety. In the evening, he became restless, jumped out of bed, shook the 'spiders' from the blanket and clothes, saw 'healthy rats and snakes' everywhere, drove away non-existent flies. Believes that he is at home, hears 'gypsy voices' outside the window, wants to run to 'drive them out of the garden'. Somatic condition: temperature - 37.8oC, marked hyperhidrosis, tremor of the whole body. What is the most likely diagnosis?

Алкогольний делірій Alcoholic delirium

Істеричний сутінковий стан Hysterical twilight state

Патологічне сп’яніння Pathological intoxication

Алкогольний галюциноз Alcoholic hallucinosis

Гострий алкогольний параноїд Acute alcoholic paranoid

15 / 200
Чотирирічний хлопчик провів цілий день на пляжі. Над вечір у дитини з’явилися: головний біль, слабкість, блювання. При об’єктивному огляді: обличчя гіперемоване, температура тіла 38,8oC, ЧД- 28/хв, ЧСС- 130/хв. Найбільш імовірною причиною такого стану є: A four-year-old boy spent the whole day on the beach. In the evening, the child developed: headache, weakness, vomiting. On objective examination: hyperemic face, temperature body 38.8oC, heart rate - 28/min, heart rate - 130/min. The most likely cause of this condition is:

Анафілактичний шок Anaphylactic shock

Сонячний удар Sunstroke

Колапс Collapse

Синкопальний стан Syncopal state

Запаморочення Dizziness

16 / 200
Дівчинка 13-ти років з виразковою хворобою дванадцятипалої кишки знаходиться на диспансерному обліку впродовж року. Яким повинен бути термін диспансеризації після загострення виразкової хвороби? A 13-year-old girl with a duodenal ulcer has been in the clinic for a year. What should be the term of the clinic after an exacerbation of the ulcer?

2 роки 2 years

5 років 5 years

3 роки 3 years

1 рік 1 year

4 роки 4 years

17 / 200
У хворого на лімфогранулематоз збільшені шийно-надключичні лімфатичні вузли справа. Загальний стан задовільний, пітливості немає, температура тіла 36,7oC. Яке лікування треба призначити? A patient with lymphogranulomatosis has enlarged cervical-supraclavicular lymph nodes on the right. The general condition is satisfactory, there is no sweating, the body temperature is 36.7oC. What treatment should be prescribed?

Монохіміотерапія Monochemotherapy

Комбінована хіміотерапія Combined chemotherapy

Комбінована хіміотерапія + радіотерапія Combined chemotherapy + radiotherapy

Монохіміотерапія + радіотерапія Monochemotherapy + radiotherapy

Радіотерапія Radiotherapy

18 / 200
Чоловік 35-ти років скаржиться на зростаючу слабкість, серцебиття, 'метелики' перед очима, запаморочення. В анамнезі: пептична виразка шлунка. Об’єктивно: шкіра бліда. В легенях -везикулярне дихання. Над верхівкою серця - систолічний шум. Ps- 100/хв., АТ100/70 мм рт.ст. Пальпаторно - незначна болючість в епігастрії. У крові: ер.- 3,2 • 1012/л, Hb- 100 г/л, КП- 0,95. Який характер анемії в даному випадку? A 35-year-old man complains of increasing weakness, palpitations, 'butterflies' before the eyes, dizziness. History: peptic ulcer of the stomach. Objectively: pale skin . In the lungs - vesicular breathing. Above the top of the heart - systolic murmur. Ps- 100/min., BP 100/70 mm Hg. On palpation - slight pain in the epigastrium. In the blood: er.- 3.2 • 1012/l, Hb - 100 g/l, CP - 0.95. What is the nature of anemia in this case?

Гемолітична анемія Hemolytic anemia

Постгеморагічна анемія Posthemorrhagic anemia

Сидеробластна анемія Sideroblastic anemia

Хронічна залізодефіцитна анемія Chronic iron deficiency anemia

Гіпопластична анемія Hypoplastic anemia

19 / 200
В харчоблок лікарні завезли знежирений кефір. На кришці фляги виставлена дата його зберігання до моменту доставки. Зазначте оптимальний термін (години) зберігання кефіру: Defatted kefir was delivered to the hospital food block. The date of its storage until the moment of delivery is displayed on the lid of the flask. Specify the optimal term (hours) for kefir storage:

20 20

40 40

50 50

60 60

36 36

20 / 200
Дайте оцінку фізичного розвитку дівчинки 12-ти років за методом си-гмальних відхилень, якщо показник ваги тіла дорівнює +0,5 сигми: Give an estimate of the physical development of a 12-year-old girl using the method of sigma deviations, if the body weight indicator is +0.5 sigma:

Високий High

Вище середнього Above Average

Середній Average

Низький Low

Нижче середнього Below Average

21 / 200
Під час медичного огляду у хворого було виявлено затримку росту, гіпогонадизм, статевий інфантилізм, відсутність вторинних статевих ознак, збільшення печінки та селезінки. Це свідчить про недостатність у харчуванні такого елементу: During the medical examination, growth retardation, hypogonadism, sexual infantilism, absence of secondary sexual characteristics, enlargement of the liver and spleen were found in the patient. This indicates insufficient nutrition of such a element:

Йод Iodine

Кальцій Calcium

Цинк Zinc

Вітамін D Vitamin D

Мідь Copper

22 / 200
Хвора 30-ти років після перебування на заробітках звернулась до дерматолога із скаргами на висипання по всьому шкірному покриву. За останні 3 місяці відмічає різке похудання, загальну слабість, постійний субфебрилітет. На предмет якого захворювання доцільно обстежити пацієнтку? A 30-year-old patient after working for a living turned to a dermatologist with complaints of a rash all over the skin. Over the past 3 months, she has noticed a sharp weight loss, general weakness, constant low fever. For what disease is it advisable to examine the patient?

Цитомегаловірусна інфекція Cytomegalovirus infection

СНІД AIDS

Сифіліс Syphilis

Туберкульоз Tuberculosis

Злоякісні новоутворення шкіри Malignant neoplasms of the skin

23 / 200
Дівчинка 12-ти років проживає у незадовільних соціально-побутових умовах. Скаржиться на періодичний переймистий біль у животі, закрепи, швидку втомлюваність. Дитина емоційно лабільна. При пальпації живота визначається болючість по ходу товстої кишки, спазмована сигмоподібна кишка. При колоноскопії змін з боку слизової оболонки товстої кишки не виявлено. Копрологічне дослідження: калові маси фрагментовані, із домішками невеликої кількості слизу, не містять решток їжі чи продуктів запалення. Яким є найбільш імовірний діагноз? A 12-year-old girl lives in unsatisfactory social and household conditions. She complains of intermittent abdominal pain, constipation, rapid fatigue. The child is emotionally unstable. When palpating the abdomen painfulness along the course of the colon, spasmodic sigmoid colon is determined. During colonoscopy, no changes in the mucous membrane of the colon were detected. Coprological examination: fecal masses are fragmented, with admixtures of a small amount of mucus, do not contain remnants of food or inflammatory products. What is the most likely diagnosis?

Хвороба Крона Crohn's disease

Синдром подразнення товстої кишки Irritable colon syndrome

Неспецифічний виразковий коліт Nonspecific ulcerative colitis

Дизентерія Dysentery

Паралітична кишкова непрохідність Paralytic intestinal obstruction

24 / 200
Дитині 7 років. Скаржиться на біль нападоподібного характеру, який виникає після психічного навантаження, вживання холодних напоїв, морозива. Після клінікоінструментального обстеження виставлений діагноз: дискінезія жовчного міхура за гіпертонічним типом. Препарати яких груп слід призначити в першу чергу для лікування? The child is 7 years old. He complains of pain of an attack-like nature, which occurs after mental stress, consumption of cold drinks, ice cream. After a clinical-instrumental examination, the diagnosis was made: dyskinesia of the gallbladder due to hypertensive by type. Drugs of which groups should be prescribed first of all for treatment?

Спазмолітики та холеретики Spasmolytics and choleretics

Антибіотики Antibiotics

Седативні та холекінетики Sedatives and cholekinetics

Антиоксиданти Antioxidants

Холеретики та холекінетики Choleretics and cholekinetics

25 / 200
Жінка 36-ти років страждає на поліноз впродовж 7-ми років. Останні 2 роки у серпнівересні (період цвітіння амброзії) пацієнтка відмічає 2-3 напади ядухи, котрі усуваються 1 дозою сальбутамолу Об’єктивно: температура -36,5oC, ЧД- 18/хв., Ps- 78/хв., АТ-115/70 мм рт.ст. Над легенями - везикулярне дихання. Тони серця звучні, ритм правильний. Застосування якого препарату дозволить найбільш ефективно запобігти нападам ядухи у критичний для хворої сезон? A 36-year-old woman has been suffering from hay fever for 7 years. For the last 2 years, in August and September (ambrosia flowering period), the patient has had 2-3 attacks of hay fever, which are eliminated by 1 dose of salbutamol Objectively: temperature -36.5oC, blood pressure - 18/min., Ps - 78/min., BP - 115/70 mm Hg. Above the lungs - vesicular breathing. Heart sounds are sonorous, the rhythm is correct . The use of which drug will allow the most effective prevention of measles attacks in a critical season for the patient?

Інгаляції інталу Intal inhalations

Прийом супрастину Taking Suprastin

Прийом теопеку Teopek reception

Інгаляції атровенту Atrovent inhalations

Інгаляції беротеку Inhalations of berotec

26 / 200
Хворий 56-ти років хворіє на інсулінонезалежний діабет. Хвороба компенсується дієтою та глюренормом. Хворий чекає на операцію з приводу панарицію. Тактика гіпоглікемізуючої терапії? A 56-year-old patient suffers from non-insulin-dependent diabetes. The disease is compensated by diet and glurenorm. The patient is waiting for an operation for panaritium. Tactics of hypoglycemic therapy?

Призначити рекомбінантний інсулін Prescribe recombinant insulin

Не змінювати терапію Do not change therapy

Призначити монокомпонентний інсулін Prescribe monocomponent insulin

Призначити манініл Assign maninil

Відмінити глюренорм Undo gluenorm

27 / 200
Чоловік 26-ти років скаржиться на спрагу, підвищене сечовиділення, загальну кволість, зменшення ваги тіла. Об’єктивно: шкіра суха, червоні щоки, дихання везикулярне. Тони серця звучні. Язик сухий. Симптомів подразнення очеревини немає. Яке дослідження є найбільш інформативним для уточнення діагнозу? A 26-year-old man complains of thirst, increased urination, general weakness, decreased body weight. Objectively: dry skin, red cheeks, vesicular breathing. Tones the heart is sound. The tongue is dry. There are no symptoms of peritoneal irritation. Which study is the most informative for clarifying the diagnosis?

Загальний аналіз крові General blood test

Загальний аналіз сечі General urinalysis

Аналіз крові на печінкові проби Blood analysis for liver samples

Аналіз сечі за Зимницьким Zymnitsky urine analysis

Аналіз крові на цукор Blood sugar analysis

28 / 200
У дитини 9-ти років виник біль у верхній третині правої гомілки, підвищилась температура до 39oC, не може стати на ногу. В анамнезі мав травму гомілки та переніс ангіну. Який найбільш імовірний діагноз? A 9-year-old child developed pain in the upper third of the right shin, the temperature rose to 39oC, he could not stand up. He had a history of shin injury and angina . What is the most likely diagnosis?

Перелом кістки Bone fracture

Злоякісна пухлина Malignant tumor

Гострий гематогенний остеомієліт Acute hematogenous osteomyelitis

Туберкульозний остеомієліт Tuberculous osteomyelitis

Гострий ревматизм Acute rheumatism

29 / 200
Жінка 32-х років скаржиться на біль у лівому підребер’ї, який з’являється через 2 години після їжі, нудоту, здуття живота, схильність до проносу. Об’єктивно: субіктеричність склер. Живіт болючий при пальпації в точці Губергриця-Скульського. Печінка - біля краю реберної дуги. У крові: амілаза - 288 ммоль/л, загальний білірубін - 20 мкмоль/л. Яке захворювання найбільш імовірно? A 32-year-old woman complains of pain in the left hypochondrium, which appears 2 hours after eating, nausea, bloating, and a tendency to diarrhea. Objectively: subicteric sclera. The abdomen is painful on palpation at the Gubergryts-Skulsky point. The liver is near the edge of the costal arch. In the blood: amylase - 288 mmol/l, total bilirubin - 20 μmol/l. What disease is most likely?

Хронічний ентероколіт Chronic enterocolitis

Хронічний гастрит Chronic gastritis

Хронічний холецистит Chronic cholecystitis

Хронічний панкреатит Chronic pancreatitis

Хронічний гепатит Chronic hepatitis

30 / 200
Хвора 50-ти років скаржиться на розпираючий біль у лівій нижній кінцівці, який посилюється при фізичному навантаженні, припухлість в ділянці гомілки та ступні. Об’єктивно: ліві гомілка та ступня пастозні, шкіра в нижній частині гомілки індурована, бронзового відтінку, розширені підшкірні вени, наявна виразка з некротичними масами. Який найбільш імовірний діагноз? A 50-year-old patient complains of excruciating pain in the left lower extremity, which worsens with physical exertion, swelling in the lower leg and foot. Objectively: left lower leg and the foot are pasty, the skin in the lower part of the leg is indurated, of a bronze hue, the subcutaneous veins are dilated, there is an ulcer with necrotic masses. What is the most likely diagnosis?

Гангрена нижньої кінцівки Gangrene of the lower extremity

Хронічна артеріальна недостатність Chronic arterial insufficiency

Гострий артеріальний тромбоз Acute arterial thrombosis

Посттромбофлебітичний синдром Postthrombophlebitic syndrome

Тромбоз глибоких вен нижніх кінцівок Thrombosis of the deep veins of the lower extremities

31 / 200
Дитина з 10-річного віку хворіє на ревматизм. Перенесла 2 атаки. Вкажіть заходи вторинної профілактики ревматизму у дитини: A child has been suffering from rheumatism since he was 10 years old. He suffered 2 attacks. Specify measures for the secondary prevention of rheumatism in a child:

Цілорічна біцилінотерапія протягом 3-х років Year-round bicillinotherapy for 3 years

Цілорічна біцилінотерапія протягом 5-ти років Year-round bicillinotherapy for 5 years

Курси нестероїдних протизапальних засобів навесні та восени Courses of nonsteroidal anti-inflammatory drugs in spring and autumn

Призначення імунокорегуючих засобів двічі на рік Prescribing immunocorrective agents twice a year

Кардіотрофіки + вітаміни навесні і восени Cardiotrophics + vitamins in spring and autumn

32 / 200
Жінка 43-х років висловлює скарги на болі у променевозап’ясткових, міжфалангових суглобах кистей і стоп, скутість рухів вранці. Хворіє 7 років. Захворювання почалося з симетричного ураження дрібних суглобів кистей і стоп. Відзначається деформація кистей і стоп, атрофія міжкісткових м’язів, набряк і запалення променевозап’ясткових суглобів; рухи у них обмежені і різко болісні. Внутрішні органи без особливостей. Який показник найбільш інформативний для встановлення діагнозу? A 43-year-old woman complains of pain in the radiocarpal, interphalangeal joints of the hands and feet, stiffness of movements in the morning. She has been ill for 7 years. The disease began with a symmetrical lesion small joints of the hands and feet. Deformation of the hands and feet, atrophy of the interosseous muscles, swelling and inflammation of the radiocarpal joints are noted; their movements are limited and sharply painful. The internal organs are without features. Which indicator is the most informative for establishing a diagnosis?

С-реактивний протеїн C-reactive protein

Лейкоцитоз Leukocytosis

Прискорена ШЗЕ Accelerated SZE

Диспротеїнемія Dysproteinemia

Ревматоїдний фактор Rheumatoid factor

33 / 200
Жінка 35-ти років скаржиться на біль у ділянці серця ('щемить, свердлить'), що виникає переважно у ранкові години в осінньо-весняний період, з іррадіацією болю в шию, спину, живіт; часте серцебиття, а також зниження загального життєвого тонусу. Виникнення цього стану не зв’язане з фізичним навантаженням. Увечері стан поліпшується. Соматичний, неврологічний статус та ЕКГ - без патології. Яка найбільш імовірна патологія зумовила таку клінічну картину? A 35-year-old woman complains of pain in the area of the heart ('pinching, drilling'), which occurs mainly in the morning hours in the autumn-spring period, with irradiation pain in the neck, back, abdomen; frequent heartbeat, as well as a decrease in general vitality. The occurrence of this condition is not associated with physical exertion. In the evening, the condition improves. Somatic, neurological status and ECG - without pathology. What is the most likely pathology that caused such a clinical picture?

Іпохондрична депресія Hypochondriac depression

Нейроциркуляторна дистонія Neurocirculatory dystonia

Неврозоподібна шизофренія Neurosis-like schizophrenia

Стенокардія спокою Angina at rest

Соматизована депресія Somatized depression

34 / 200
Батьки хлопчика 7-ми років звернулись до лікаря зі скаргами на те, що у дитини останні 2-3 тижні спостерігається поліурія, спрага, втрата маси тіла до 4 кг. Об’єктивно: шкіра бліда, суха, на щоках рум’янець, губи і язик сухі, тургор тканин знижений, запах ацетону із рота. Який діагноз найбільш імовірний? Parents of a 7-year-old boy turned to the doctor with complaints that the child has been experiencing polyuria, thirst, and weight loss of up to 4 kg for the past 2-3 weeks Objectively: the skin is pale, dry, the cheeks are flushed, the lips and tongue are dry, the turgor of tissues is reduced, the smell of acetone from the mouth. What is the most likely diagnosis?

Цукровий діабет Diabetes

Нирковий діабет Renal diabetes

Нецукровий діабет Diabetes insipidus

Ацетонемічний синдром Acetonemic syndrome

Синдром Альпорта Alport Syndrome

35 / 200
Студент звернувся до лікаря зі скаргами на безсоння, дратівливість, серцебиття. Мати хвора на гіпертонічну хворобу. При обстеженні: пітливість долоней, дрібний тремор рук, стійкий червоний дермографізм. У легенях - везикулярне дихання, межі серця в нормі, серцева діяльність ритмічна, 80/хв., АТ-150/65 мм рт.ст. Який найбільш імовірний діагноз? The student turned to the doctor with complaints of insomnia, irritability, palpitations. The mother has hypertension. During the examination: sweating of the palms, small tremor of the hands, persistent red dermographism. In the lungs - vesicular breathing, heart boundaries are normal, cardiac activity is rhythmic, 80/min., BP-150/65 mm Hg. What is the most likely diagnosis?

ВСД за гіпертонічним типом VSD by hypertensive type

Гіпертонічна хвороба Hypertensive disease

Феохромоцитома Pheochromocytoma

Вузликовий періартеріїт Nodular periarteritis

Тиреотоксикоз Thyrotoxicosis

36 / 200
У районі, який обслуговує дитяча поліклініка, щепленням підлягало 4800 дітей та підлітків, оглянуто дітей 4800, зроблені профілактичні щеплення 4320 особам, у 480 дітей виявлені тимчасові медичні протипоказання. Яка величина показника 'Повнота охоплення профілактичними щепленнями дітей та підлітків, що підлягають щепленням'у цьому районі? In the area served by the children's polyclinic, 4,800 children and teenagers were vaccinated, 4,800 children were examined, 4,320 people were given preventive vaccinations, and temporary medical contraindications were found in 480 children. What is the value of the indicator 'Completeness of preventive vaccination coverage of children and adolescents subject to vaccination' in this area?

90% 90%

60% 60%

100% 100%

80% 80%

70% 70%

37 / 200
Хворий 20-ти років скаржиться на сильний головний 6іль, двоїння в очах, загальну слабкість, підвищення температури тіла, дратiвливiсть. Об’єктивно: температура тіла 38,1oC, в контакт вступає неохоче, болісно реагує на подразники. Птоз лівої повіки, розбіжна косоокість, анізокорія S>D. Виражений менінгеальний синдром. При люмбальній пункції ліквор витікав під тиском 300 мм вод.ст., прозорий, з легкою опалесценцією, через добу випала фібринозна плівка. Білок -1,4 г/л, лімфоцити - 600/3 в мм3, цукор - 0,3 ммоль/л. Який попередній діагноз слід поставити хворому? A 20-year-old patient complains of a severe headache, double vision, general weakness, increased body temperature, irritability. Objectively: body temperature 38.1oC , comes into contact reluctantly, reacts painfully to stimuli. Ptosis of the left eyelid, divergent strabismus, anisocoria S>D. Pronounced meningeal syndrome. During a lumbar puncture, cerebrospinal fluid flowed out under a pressure of 300 mm Hg, transparent, with a slight opalescence, after a day it fell out fibrinous film. Protein - 1.4 g/l, lymphocytes - 600/3 in mm3, sugar - 0.3 mmol/l. What preliminary diagnosis should be given to the patient?

Сифілітичний менінгіт Syphilitic meningitis

Лімфоцитарний менінгіт Армстронга Armstrong lymphocytic meningitis

Менінгококовий менінгіт Meningococcal meningitis

Туберкульозний менінгіт Tuberculous meningitis

Паротитний менінгіт Mumps meningitis

38 / 200
У чоловіка через 16 годин після вживання в їжу в’яленої риби і грибів виникло блювання, з’явилися слабкість, задишка, сухість у роті, запаморочення. Затримка стулу. На наступний день хворий госпіталізований у важкому стані з частим пульсом при нормальній температурі тіла, зі скаргами на поганий зір і двоїння в очах. Який найбільш імовірний діагноз? The man vomited 16 hours after eating dried fish and mushrooms, weakness, shortness of breath, dry mouth, dizziness appeared. Stool retention . The next day, the patient is hospitalized in serious condition with a frequent pulse at a normal body temperature, with complaints of poor vision and double vision. What is the most likely diagnosis?

Опісторхоз Opistorchosis

Отруєння грибами Mushroom poisoning

Ботулізм Botulism

Стафілококовий токсикоз Staphylococcal toxicosis

Харчова токсикоінфекція Food poisoning

39 / 200
Голівка новонародженого має доліхоцефалічну форму, витягнута спереду назад. Під час огляду голівки на потиличній частині визначається пологова пухлина, розташована на середині між великим і малим тім’ячком. При якому передлежанні голівки плоду відбулися пологи? The head of a newborn has a dolichocephalic shape, elongated from front to back. During the examination of the head on the occipital part, a birth tumor is determined, located in the middle between the large and small parietal. to which presentation of the fetal head did the birth take place?

Передній вид потиличного передлежання Anterior view of occipital presentation

Передньо-тім’яне передлежання Anterior-parietal presentation

Лобне передлежання Frontal presentation

Задній вид потиличного передлежання Posterior view of occipital presentation

Лицеве передлежання Face-to-face presentation

40 / 200
У хворої з підозрою на феохромоцитому, у період між нападами, артеріальний тиск в межах норми, відзначається тенденція до тахікардії. У сечі патології немає. Вирішено провести провокаційну пробу з гістаміном. Який препарат слід підготувати для надання екстреної допомоги при позитивному результаті проби? A patient with a suspicion of pheochromocytoma, in the period between attacks, blood pressure is within the normal range, there is a tendency to tachycardia. There is no pathology in the urine. It was decided to conduct a provocative test with histamine. What drug should be prepared for providing emergency care in case of a positive test result?

Піпольфен Pipolfen

Ніфедипін Nifedipine

Преднізолон Prednisone

Фентоламін Phentolamine

Мезатон Mesaton

41 / 200
Хворий 54-х років скаржиться на часте болісне сечовипускання, озноби, підвищення температури тіла до 38oC. У сечі: білок - 0,33 г/л, лейкоцити до 5060 у п/з, еритроцити - 5-8 у п/з, грамнегативні палички. Який з представлених антибактеріальних препаратів найкращий у цьому випадку? A 54-year-old patient complains of frequent painful urination, chills, an increase in body temperature up to 38oC. In the urine: protein - 0.33 g/l, leukocytes up to 5060 in p/z, erythrocytes - 5-8 in p/z, gram-negative rods. Which of the presented antibacterial drugs is the best in this case?

Тетрациклін Tetracycline

Оксацилін Oxacillin

Еритроміцин Erythromycin

Цепорін Tseporin

Ципрофлоксацин Ciprofloxacin

42 / 200
Терапевти міської лікарні протягом дня обслуговують населення в 3-х ланках: прийом, дільниця, стаціонар. Через кожні 2 роки на 3-4 місяці лікарі-терапевти направляються в стаціонар. За якою системою дільничності працюють лікарі? Therapists of the city hospital serve the population during the day in 3 sections: reception, ward, inpatient. Every 2 years, for 3-4 months, the therapists are sent to inpatient hospital. Under what district system do doctors work?

Територіальна дільничність Territorial subdivision

Триланкова Three-link

Дволанкова Dvolankova

Цехова дільничність Workshop area

Чергування Rotation

43 / 200
У дитини під час огляду після народження відзначена брахіцефалічна форм черепа, деформація вушних раковин у вигляді виступаючої протизавитки, епікант, косий розріз очей, вкорочення мізинця, двобічна поперечна складка на долоні. При аускультації серця вислуховується грубий систолічний шум з максимумом у 3-4 міжребер’ях по лівому краю груднини. Яка вроджена вада серця найбільш часто супроводжує цю патологію? During the examination after birth, the child was noted to have a brachycephalic form of the skull, deformation of the auricles in the form of a protruding anti-coil, epicanth, oblique incision of the eyes, shortening of the little finger, bilateral transverse fold on palms. When auscultating the heart, a coarse systolic murmur is heard with a maximum in 3-4 intercostal spaces on the left side of the sternum. What congenital heart defect most often accompanies this pathology?

Стеноз легеневої артерії Pulmonary artery stenosis

Коарктація аорти Coarctation of the aorta

Вади Фало Vadi Falo

Фiброеластоз Fibroelastosis

Відкрита артерiальна протока Open ductus arteriosus

44 / 200
Жiнка 40 -ка років госпіталізована в клініку в зв’язку з загостренням хронічного обструктивного бронхіту. Скаржиться на задишку при фізичному навантаженні, кашель з виділенням мокроти, набряки нижніх кінцівок. Об’єктивно: температура - 37,2oC, ЧД-24/хв., Ps- 90/хв., ритмічний, АТ- 110/70 мм рт.ст. Акцент II тону над легеневою артерією. Пальпується збільшена на 4 см печінка. Які зміни ЕКГ найбільш імовірні у хворої? A 40-year-old woman was hospitalized due to an exacerbation of chronic obstructive bronchitis. She complains of shortness of breath during physical exertion, cough with sputum, swelling of the lower extremities Objectively: temperature - 37.2oC, BH-24/min., Ps- 90/min., rhythmic, BP- 110/70 mm Hg. Emphasis of the II tone over the pulmonary artery. Liver enlarged by 4 cm is palpated What ECG changes are most likely in the patient?

Гіпертрофія лівого шлуночка Left ventricular hypertrophy

Миготлива аритмія Atrial fibrillation

Патологічний зубець Q Pathological Q wave

Гіпертрофія правого шлуночка Right ventricular hypertrophy

Блокада лівої ніжки пучка Гіса Blockade of the left leg of the bundle of His

45 / 200
Головний лікар поліклініки дав завдання дільничому лікарю визначити патологічну ураженість населення N-ою хворобою на дільниці. За яким документом визначається патологічна ураженість населення на хвороби на лікарській дільниці? The head doctor of the polyclinic gave the precinct doctor the task of determining the pathological susceptibility of the population to the Nth disease in the precinct. According to what document is the pathological susceptibility of the population to diseases in the medical precinct determined?

Статистичні талони (+) Statistical vouchers (+)

Статистичні талони (+) та (-) Statistical coupons (+) and (-)

Журнал профоглядів Log of professional views

Статистичні талони (-) Statistical coupons (-)

Талони на прийом до лікаря Coupons for doctor's appointments

46 / 200
Дільничим лікарем після тривалого диспансерного спостереження прийнято рішення щодо переведення хворого на інвалідність. Хто буде направляти хворого на медикосоціальну експертну комісію? After a long dispensary observation, the district doctor made a decision to transfer the patient to disability. Who will refer the patient to the medical and social expert commission?

Лікарська консультативна комісія Medical Advisory Commission

Лікар вузької спеціальності Doctor of a narrow specialty

Завідувач відділенням поліклініки Head of the department of the polyclinic

Дільничий лікар District doctor

Завідувач стаціонарного відділення Head of inpatient department

47 / 200
Чоловік 44-х років надійшов до інфекційного відділення з діагнозом: фолікулярна ангіна. Температура тіла -38,6oC, шкіра та слизові звичайного кольору. Ps- і02/хв., АТ- 130/70 мм рт.ст. У крові: ер.- 2,7-1012/л, Hb- 90 г/л, Кп- 1,0; лейк.- 38-109 /л, бласти - 68%, п- 2%, с - 14%, л- 14%, м- 2%, ШЗЕ- 46 мм/год. Яке захворювання слід запідозрити у хворого? A 44-year-old man was admitted to the infectious department with a diagnosis of follicular tonsillitis. Body temperature -38.6oC, skin and mucous membranes of normal color. Ps- and 02/min ., blood pressure - 130/70 mm Hg. In the blood: er. - 2.7-1012/l, Hb - 90 g/l, Kp - 1.0; leuk. - 38-109 /l, blasts - 68%, p - 2%, c - 14%, l - 14%, m - 2%, SZE - 46 mm/h. What disease should be suspected in the patient?

Хронічний мієлолейкоз Chronic myelogenous leukemia

Гострий агранулоцитоз Acute agranulocytosis

Гострий лейкоз Acute leukemia

Хронічний лімфолейкоз Chronic lymphocytic leukemia

Лейкемоїдна реакція Leukemoid reaction

48 / 200
Чоловік 55-ти років надійшов до клініки в зв’язку з нападом ниркової кольки, яка періодично повторюється протягом року. Об’єктивно: в ділянці вушних раковин та правого ліктьового суглоба знаходяться вузликові утворення, вкриті тонкою блискучою шкірою. Ps- 88/хв. АТ-170/100 мм рт.ст. Позитивний симптом Пастернацького з обох боків. Хворому призначено обстеження. Вивчення якого лабораторного показника найбільш доцільно для встановлення діагнозу? A 55-year-old man came to the clinic due to an attack of renal colic, which periodically recurs during the year. Objectively: in the area of the auricles and right of the elbow joint, there are nodular formations covered with thin, shiny skin. Ps- 88/min. BP-170/100 mm Hg. Positive Pasternacki's symptom on both sides. The patient is scheduled for an examination. Which laboratory indicator is the most appropriate for establishing a diagnosis?

Осад сечі Urine sediment

Ревматоїдний фактор Rheumatoid factor

Молочна кислота Lactic acid

ШЗЕ ШЭ

Сечова кислота Uric acid

49 / 200
Чоловік 55-ти років впродовж 1,5 років спостерігається з приводу вірусного цирозу печінки з явищами портальної гіпертензії. За останній місяць посилилась слабкість, з’явилося блювання кольору 'кавової гущі'. При фіброгастродуоденоскопії виявлена кровотеча з розширених вен стравоходу. Який препарат необхідно використати для зниження тиску в ворітній вені? A 55-year-old man has been observed for 1.5 years for viral cirrhosis of the liver with symptoms of portal hypertension. Over the past month, weakness has increased, vomiting of color has appeared' of coffee grounds'. Fibrogastroduodenoscopy revealed bleeding from dilated veins of the esophagus. What drug should be used to reduce pressure in the portal vein?

Вазопресин Vasopressin

Резерпін Reserpin

Глюконат кальцію Calcium gluconate

Фуросемід Furosemide

Дицінон Dizinon

50 / 200
Жінка 58-ми років скаржиться на безпричинну появу синців, слабкість, кровоточивість ясен, запаморочення. Об’єктивно: слизові оболонки та шкірні покриви бліді, з численними крововиливами різної давнини. Лімфатичні вузли не збільшені. Ps- 100/хв., АТ-110/70 мм рт.ст. З боку внутрішніх органів змін не виявлено. У крові: ер.-3,0-1012/л, НЬ92 г/л, КП- 0,9, анізоцитоз, пойкілоцитоз, лейк.- 10-109 /л, е-2%, п- 12%, с- 68%, л- 11%, м7%, ШЗЕ-12 мм/год. Додаткове визначення якого лабораторного показника найбільш доцільне для встановлення діагнозу? A 58-year-old woman complains of unexplained bruising, weakness, bleeding gums, dizziness. Objectively: mucous membranes and skin are pale, with numerous hemorrhages of various of ancient times. Lymph nodes are not enlarged. Ps- 100/min., BP-110/70 mm Hg. No changes were detected on the part of internal organs. Blood: ER-3.0-1012/l, Hb92 g/l . Additional determination of which laboratory indicator is the most appropriate for establishing a diagnosis?

Час згортання крові Blood clotting time

Осмотична резистентність еритроцитів Osmotic resistance of erythrocytes

Ретикулоцити Reticulocytes

Фі6риногєн Phy6rinogen

Тромбоцити Platelets

51 / 200
Хворий 25-ти років надійшов до інфекційного відділєння на 3-й день захворювання зі скаргами на головний біль, біль у попереку, литкових м’язах, високу гарячку, озноб. Стан середньо-важкий. Склери іктеричні. Зів гіперемований. Язик сухий, обкладений сухим коричневим нальотом. Живіт здутий. Печінка +2 см. Селезінка не збільшена. Пальпація м’язів, особливо литкових, болісна. Сеча темна. Кал звичайного кольору. Який найбільш імовірний діагноз? A 25-year-old patient came to the infectious disease department on the 3rd day of the disease with complaints of headache, pain in the lower back, calf muscles, high fever, Chills. Condition moderate to severe. Scleres icteric. Pharynx hyperemic. Tongue dry, coated with a dry brown coating. Abdomen distended. Liver +2 cm. Spleen not enlarged. Palpation of muscles, especially calves, painful. Urine dark. Stool of normal color. What is the most likely diagnosis?

Малярія Malaria

Лептоспіроз Leptospirosis

Інфекційний мононуклеоз Infectious mononucleosis

Вірусний гепатит А Viral hepatitis A

Ієрсиніоз Yersiniosis

52 / 200
на раптовий початок хвороби: лихоманка до 39,0o 52. C, неправильного типу, з гострим болем у грудях. Харкотиння з гнилісним запахом та домішками крові, до 400 мл за добу. При перкусії: над ураженою ділянкою вкорочення перкуторного звуку, посилене голосове тремтіння. У харкотинні був виділений анаеробний стрептокок. Яку хворобу можна запідозрити в першу чергу? for the sudden onset of the disease: fever up to 39.0o 52. C, of the wrong type, with sharp pain in the chest. Sputum with a putrid smell and impurities of blood, up to 400 ml per day. Percussion: shortening of the percussion sound over the affected area, increased vocal tremor. Anaerobic streptococcus was isolated in the sputum. What disease can be suspected first of all?

Абсцес легені Lung abscess

Гангрена легені Gangrene of the lungs

Бронхоектатична хвороба Bronchoectatic disease

Абсцедуюча пневмонія Abscessing pneumonia

Туберкульоз Tuberculosis

53 / 200
У хворого 50-ти років раптово виникли біль у потиличній ділянці, блювання. Об’єктивно: сопор, гіперемія шкіри обличчя, АТ- 210/і20 мм рт.ст., Ps-60/хв, напружений, температура тіла -37,8oC. Горизонтальний ністагм. Виражені рефлекси орального автоматизму. Сухожилкові рефлекси рівномірні. Ригідність потиличних м’язів, двобічний симптом Керніга. Який попередній діагноз? A 50-year-old patient suddenly developed pain in the back of the head, vomiting. Objectively: sopor, hyperemia of the skin of the face, blood pressure - 210/i20 mm Hg. Art., Ps-60/min, tense, body temperature -37.8oC. Horizontal nystagmus. Pronounced reflexes of oral automatism. Tendon reflexes are even. Rigidity of occipital muscles, bilateral Kernig sign. What is the previous diagnosis?

Менінгококовий менінгіт Meningococcal meningitis

Субарахноїдальний крововилив Subarachnoid hemorrhage

Геморагічний паренхіматозний інсульт Hemorrhagic parenchymal stroke

Гостра гіпертонічна енцефалопатія Acute hypertensive encephalopathy

Субдуральна гематома Subdural hematoma

54 / 200
-ка років скаржиться на підвищення температури до 39o 54. C, кашель з виділенням харкотиння та домішками крові, задишку, загальну слабкість, герпетичне висипання на губах. Об’єктивно: ЧД- 32/хв. Справа під лопаткою - посилення голосового тремтіння, там же - притуплення перкуторного звуку. Аускультативно: бронхіальне дихання. У крові: лейк.- 14 • 109 /л, ШЗЕ- 35 мм/год. Який попередній діагноз? year-old complains of an increase in temperature to 39o 54. C, cough with sputum discharge and blood impurities, shortness of breath, general weakness, herpetic rash on the lips. Ob' Objectively: ChD- 32/min. On the right under the scapula - increased vocal tremor, in the same place - dulling of the percussion sound. Auscultation: bronchial breathing. In the blood: leuk.- 14 • 109 /l, SZE- 35 mm/h. What is the previous diagnosis ?

Крупозна правобічна пневмонія Croupous right-sided pneumonia

Рак легені Lung cancer

Ексудативний плеврит Exudative pleurisy

Кавернозний туберкульоз правої легені Cavernous tuberculosis of the right lung

Вогнищева правобічна пневмонія Focal right-sided pneumonia

55 / 200
У доношеної дитини віком 6 днів на різних ділянках шкіри виявляються еритема, мляві пухирі, ерозивні поверхні, тріщини, лущення епідермісу, які виглядають ніби після ошпарення окропом. Виявлено позитивний симптом Нікольського. Загальний стан дитини важкий. Виражений неспокій, гіперестезія, фебрильна температура. Який найбільш імовірний діагноз у цьому випадку? A 6-day-old full-term child has erythema, flaccid blisters, erosive surfaces, cracks, flaking of the epidermis, which look as if after being scalded with boiling water, on various areas of the skin. Positive Nikolsky's symptom. The general condition of the child is severe. Pronounced restlessness, hyperesthesia, febrile temperature. What is the most likely diagnosis in this case?

Пухирчатка новонародженого Neonatal pemphigus

Флегмона новонародженого Phlegmon of a newborn

Псевдофурункульоз Фігнера Pseudofurunculosis Figner

Ексфоліативний дерматит Ріттера Ritter's exfoliative dermatitis

Епідермоліз Epidermolysis

56 / 200
Хворому було виконано резекцію шлунка. Під час операції ліва верхня кінцівка хворого була відведена і зафіксована до операційного столу для виконання анестезіологічного забезпечення. В післяопераційному періоді у хворого з’явились порушення функції верхньої кінцівки у вигляді 'звисаючої кисті'. Пошкодження якого нерва привело до виникнення цього симптому? The patient underwent gastric resection. During the operation, the patient's left upper extremity was removed and fixed to the operating table for anesthesia. In the postoperative period, the patient had dysfunction of the upper extremity in the form of a 'hanging hand'. Damage to which nerve led to the emergence of this symptom?

Серединний нерв Median nerve

М’язово-шкірний нерв Musculocutaneous nerve

Пахвовий нерв Axillary nerve

Ліктьовий нерв Ulnar nerve

Променевий нерв Radiant nerve

57 / 200
Жінка 37-ми років звернулась до лікаря з приводу загострення хронічного гепатиту. У крові знайдено підвищення рівня непрямого білірубіну, АсАТ, АлАТ та зниження рівня альбуміну i протромбіну Який з патологічних процесів найбільш імовірно обумовив ці зміни? A 37-year-old woman consulted a doctor about an exacerbation of chronic hepatitis. An increase in the level of indirect bilirubin, AST, ALT and a decrease in the level of albumin and prothrombin were found in the blood. Which of the pathological processes most likely caused these changes?

Портальна гіпертензія Portal hypertension

Холестаз Cholestasis

Порушення гемостазу Disruption of hemostasis

Цитоліз Cytolysis

Гіперспленізм Hypersplenism

58 / 200
Жінка 63-х років скаржиться на загальну слабкість, почуття тягаря, тиснення, переповнення в епігастрії, нудоту, відрижку після їжі. Хворіє біля 15 років. Об’єктивно: температура - 36,4oC, ЧД- 20/хв, Ps- 88/хв, АТ-115/75 мм рт.ст. Шкіра та слизові бліді. У крові: ер.-2,0-1012/л, НЬ- 100 г/л. Виявлені антитіла до обкладинкових клітин шлунка. Яка найбільш імовірна причина розвитку анемічного синдрому в даної хворої? A 63-year-old woman complains of general weakness, a feeling of heaviness, pressure, fullness in the epigastrium, nausea, belching after eating. She has been ill for about 15 years. Objectively : temperature - 36.4oC, BH - 20/min, Ps - 88/min, BP - 115/75 mm Hg. Skin and mucous membranes are pale. Blood: er.-2.0-1012/l, Hb- 100 g/l. Antibodies to the lining cells of the stomach were detected. What is the most likely reason for the development of anemic syndrome in this patient?

Порушення всмоктування заліза Iron malabsorption

Продукція антитіл до внутрішнього фактору Production of antibodies to intrinsic factor

Порушення синтезу гемоглобіну Disruption of hemoglobin synthesis

Підвищена витрата заліза Increased consumption of iron

Порушення синтезу еритропоетину Disruption of erythropoietin synthesis

59 / 200
Випадок сімейного харчового отруєння характеризувався класичною клінічною картиною ботулізму. Напередодні всі хворі вживали в їжу яєчню, борщ, котлети, варену ковбасу з консервованою кабачковою ікрою, абрикосове варення, морозиво. Який продукт найбільш імовірно викликав отруєння? The case of family food poisoning was characterized by the classic clinical picture of botulism. The day before, all patients ate scrambled eggs, borscht, cutlets, boiled sausage with canned zucchini caviar, apricot jam, ice cream . Which product most likely caused the poisoning?

Варена ковбаса Boiled sausage

Варення Jam

Морозиво Ice cream

Котлети Cutlets

Консервована кабачкова ікра Canned Zucchini Caviar

60 / 200
Жінка 42-х років страждає на мікронодулярний криптогенний цироз печінки. Протягом останнього тижня стан погіршився: з’явились судоми, затьмарення свідомості, посилилась жовтяниця. Виконання якого дослідження може пояснити причину погіршення стану? A 42-year-old woman suffers from micronodular cryptogenic cirrhosis of the liver. During the last week, the condition worsened: convulsions, blackout of consciousness appeared, and jaundice increased. What research can explain the reason for the worsening condition?

Визначення АлАТ та АсАТ Definition of AlAT and AsAT

Визначення аміаку сироватки Serum ammonia determination

Визначення рівня лужної фосфатази Alkaline phosphatase level determination

Визначення ефірів холестерину Determination of cholesterol esters

Визначення вмісту α-фетопротеїну Determination of α-fetoprotein content

61 / 200
У дівчинки 7-ми років скарги на слабкість, підвищену втомлюваність, підвищення температури тіла до 38oC, малу кількість виділеної за добу сечі, кольору 'м’ясних помиїв'. Об’єктивно: блідість шкіри, набряк обличчя, рук, ніг, біль у попереку. У крові: ер.-2,7-1012/л, НЬ- 90 г/л, лейк.- 17-109 /л, е.-10%, п.- 4%, с.- 60%, л.- 16%, м.- 10%, ШЗЕ- 30 мм/год. В сечі: лейкоц.- 15 в п/з, еритр.- 30 в п/з, гіалінові циліндри - 8-10 в п/з, білок - 4 г/л. Холестерин в крові - 8 ммоль/л, загальний білок - 43 г/л. Який провідний механізм розвитку набряків? A 7-year-old girl complains of weakness, increased fatigue, an increase in body temperature up to 38oC, a small amount of urine released per day, the color of 'meat slops' Objectively: paleness of the skin, swelling of the face, hands, feet, pain in the lower back. In the blood: er.-2.7-1012/l, Hb- 90 g/l, leuk.- 17-109 /l, e .-10%, p.- 4%, s.- 60%, l.- 16%, m.- 10%, SZE- 30 mm/h In urine: leukocyt.- 15 in p/z, erythr. - 30 in p/z, hyaline cylinders - 8-10 in p/z, protein - 4 g/l. Cholesterol in the blood - 8 mmol/l, total protein - 43 g/l. What is the leading mechanism of edema development?

Зниження онкотичного тиску крові Reduction of blood oncotic pressure

Зниження осмотичного тиску крові Decrease in blood osmotic pressure

Гіперальдостеронізм Hyperaldosteronism

Дизелектролітні порушення Dielectrolyte violations

Порушення серцевої діяльності Cardiac disorders

62 / 200
У 3-х денної доношеної новонародженої дитини спостерігається рівень непрямого білірубіну - 345 мкмоль/л, погодинний приріст - 6,8 мкм/л. Стан дитини важкий: зниження рефлексів, гіпотонія м’язів, тремор кінцівок. Кров дитини та матері несумісна за Rh-фактором. Який метод лікування найбільш ефективний? A 3-day-old full-term newborn baby has an indirect bilirubin level of 345 μmol/l, an hourly increase of 6.8 μm/l. The child's condition is serious: a decrease of reflexes, hypotonia of muscles, tremor of the limbs. The blood of the child and the mother are incompatible according to the Rh factor. What method of treatment is the most effective?

Фототерапія Phototherapy

Прийом фенобарбіталу Reception of Phenobarbital

Замінне переливання одногрупної та Rh-сумісної крові Replacement transfusion of monotype and Rh-compatible blood

Гемосорбція Hemosorption

Кортикостероїдна терапія Corticosteroid therapy

63 / 200
Хвора 32-х років скаржиться на 'душевний біль', поганий настрій, відсутність апетиту, безсоння. Вказані симптоми з’явилися поступово впродовж 3-х місяців без видимої причини. У психічному статусі: на питання відповідає тихим голосом, обличчя сумне, міміка скорботна, рухи уповільнені, емоційно пригнічена, темп мислення уповільнений. Стан хворої поліпшується ввечері і погіршується вранці. Який передбачуваний діагноз? A 32-year-old patient complains of 'mental pain', bad mood, lack of appetite, insomnia. These symptoms appeared gradually over 3 months without apparent reasons. In the mental status: questions are answered in a low voice, the face is sad, facial expressions are sad, movements are slowed down, emotionally depressed, the pace of thinking is slowed down. The patient's condition improves in the evening and worsens in the morning. What is the expected diagnosis?

Циклотимія Cyclothymia

Інволюційна меланхолiя Involutional Melancholy

Маніакально-депресивний психоз: депресивна фаза Manic-depressive psychosis: depressive phase

Реактивна депресія Reactive Depression

Депресивний синдром внаслідок органічного ураження центральної нервової системи Depressive syndrome due to organic damage to the central nervous system

64 / 200
Чоловік 57-ми років відзначає серцебиття, пітливість, порушення сну, наростаючу слабість, втрату ваги. Лікувався з приводу ІХС, без ефекту. Об’єктивно: температура 36,8oC, ЧСС-128/хв., Ps- 112/хв., аритмічний, АТ-160/70 мм рт.ст. Шкіра тепла, волога. Тремор пальців витягнутих рук. Тони серця посилені, систолічний шум над верхівкою. Щитоподібна залоза не пальпується. Яке з перерахованих досліджень найбільш важливе для уточнення діагнозу? A 57-year-old man reports palpitations, sweating, sleep disturbances, increasing weakness, weight loss. He was treated for coronary artery disease, without effect. Objectively: temperature 36 ,8oC, heart rate-128/min., Ps- 112/min., arrhythmic, BP-160/70 mm Hg. Skin is warm, moist. Tremor of fingers of outstretched hands. Heart tones are increased, systolic murmur over the apex. Thyroid gland is not palpable. Which of the listed studies is the most important for clarifying the diagnosis?

Дослідження рівня тиреоїдних гормонів у крові Research of the level of thyroid hormones in the blood

Ліпідний спектр крові Blood lipid spectrum

Ультразвукове дослідження серця Ultrasound examination of the heart

Ультразвукове дослідження щитоподібної залози Ultrasound examination of the thyroid gland

Проба з дозованим фізичним навантаженням Test with dosed physical load

65 / 200
Жінка 40-ка років, що страждає на комбіновану мітральну ваду з переважанням стенозу, скаржиться на задишку, напади ядухи вночі, перебої у роботі серця. У теперішній час не може виконувати легку домашню роботу. Яка оптимальна тактика ведення хворої? A 40-year-old woman suffering from combined mitral valve disease with predominance of stenosis, complains of shortness of breath, attacks of dyspnea at night, heart failure. At present, no can perform light housework. What is the optimal tactic of managing the patient?

Лікування серцевої недостатності Heart failure treatment

Проведення антиаритмічної терапії Conducting antiarrhythmic therapy

Проведення мітральної комісуротомії Performing mitral commissurotomy

Призначення протиревматичної терапії Prescription of antirheumatic therapy

Імплантація штучного клапана Artificial valve implantation

66 / 200
Хворий 25-ти років скаржиться на біль у ділянці серця ниючого характеру впродовж 10-ти днів, задишку при незначному фізичному навантаженні, серцебиття. Захворів 2 тижні тому після респіраторної інфекції. Об’єктивно: акроціаноз, АТ- 90/75 мм рт.ст., Ps-96/хв. Межі серця зміщені вліво і вправо. Тони серця ослаблені, тричленний ритм, систолічний шум на верхівці. ЕКГ: ритм синусовий, повна блокада лівої ніжки пучка Гіса. Який найбільш імовірний діагноз? A 25-year-old patient complains of pain in the area of the heart of an aching nature for 10 days, shortness of breath with slight physical exertion, palpitations. He became ill 2 weeks ago after respiratory infections. Objectively: acrocyanosis, blood pressure - 90/75 mm Hg, Ps - 96/min. The borders of the heart are shifted to the left and right. Heart sounds are weakened, three-part rhythm, systolic murmur at the apex. ECG: sinus rhythm, full left bundle branch block. What is the most likely diagnosis?

Інфекційний ендокардит Infective endocarditis

Ексудативний перикардит Exudative pericarditis

Вегето-судинна дистонія Vegeto-vascular dystonia

Міокардитичний кардіосклероз Myocardial cardiosclerosis

Інфекційно-алергічний міокардит Infectious-allergic myocarditis

67 / 200
Хвора 41-го року звернулася до лікаря на другий день захворювання зі скаргами на загальну слабкість, підвищення температури тіла до 38 — 39oC, рідкі випорожнення до 4-6 разів на добу зеленуватого кольору зі слизом, повторне блювання, біль у надчеревній ділянці та біля пупка. Об’єктивно: стан середньої важкості. Шкіра бліда. Ps- 92/хв., АТ-105-75 мм рт.ст. Язик обкладений. Живіт при пальпації болючий в надчеревній і правій здухвинній ділянках. Яке з обстежень найбільш імовірно підтвердить діагноз? A 41-year-old patient turned to the doctor on the second day of the illness with complaints of general weakness, an increase in body temperature to 38-39oC, loose stools up to 4-6 times per day greenish color with mucus, repeated vomiting, pain in the epigastric area and near the navel. Objectively: the condition is of medium severity. The skin is pale. Ps- 92/min., BP-105-75 mm Hg. Tongue coated. Stomach on palpation, it is painful in the epigastric and right iliac regions. Which of the examinations is most likely to confirm the diagnosis?

Вірусологічні дослідження випорожнень Stool virological studies

Бактеріологічне дослідження випорожнень Bacteriological examination of feces

Ректороманоскопія Rectoromanoscopy

УЗД органів черевної порожнини Ultrasound of abdominal organs

Копрологічне дослідження випорожнень Coprological examination of stools

68 / 200
Дитина знаходиться на стаціонарному лікуванні з приводу гострої стафілококової деструкції правої легені. Раптово з’явилися гострий біль у правій половині грудної клітки, задишка, ціаноз. Права половина грудної клітки відстає в акті дихання. Перкуторно справа внизу - тупість, в верхніх відділах - коробковий звук. Межі відносної серцевої тупості зміщені вліво. Яке ускладнення найбільш імовірно виникло? The child is under inpatient treatment for acute staphylococcal destruction of the right lung. Acute pain in the right half of the chest suddenly appeared, shortness of breath, cyanosis. The right half of the chest lags behind in the act of breathing. Percussion in the lower right - dullness, in the upper parts - a box sound. The limits of relative cardiac dullness are shifted to the left. What complication most likely occurred?

Емпієма плеври Empyema of the pleura

Піопневмоторакс справа Pyopneumothorax on the right

Спонтанний пневмоторакс Spontaneous pneumothorax

Абсцес правої легені Abscess of the right lung

Ексудативний плеврит Exudative pleurisy

69 / 200
Хворому 29-ти років проведена первинна хірургічна обробка розчавленої рани верхньої третини лівого стегна. Через 2 дні стан хворого погіршився: при огляді рана з незначними виділеннями, м’язи сіруватого кольору, кінцівка набрякла, шкіра холодна, вкрита синюшними плямами, при пальпації визначається крепітація. Свідомість запаморочена. Температура тіла 40,3oC. Яке ускладнення виникло у хворого? A 29-year-old patient underwent primary surgical treatment of a crushed wound in the upper third of the left thigh. After 2 days, the patient's condition worsened: upon examination of the wound with minor discharge, muscles grayish in color, the limb is swollen, the skin is cold, covered with bluish spots, palpation reveals crepitus. Consciousness is dizzy. Body temperature is 40.3oC. What complication did the patient have?

Бешиха Beshikha

Флегмона стегна Phlegmon of the hip

Артерїальний тромбоз Arterial thrombosis

Анаеробна газова гангрена Anaerobic gas gangrene

Правець Tetanus

70 / 200
Фотохронометражне дослїдження прийому хворих лїкарями-терапевтами поліклініки №1 у 1997 роцї показало, що на підготовку і ознайомлення з медичною карткою витрачається 10,6%, на опит - 15,1%, на огляд і обстеження - 35,9%, на інші елементи роботи -38,4% загального часу прийому одного хворого. За допомогою якого вигляду діаграм можна наочно проілюструвати результати дослідження? A photo-chronometric study of the reception of patients by therapists of Polyclinic No. 1 in 1997 showed that 10.6% was spent on preparing and familiarizing with the medical card, 15% on the interview .1%, for examination and examination - 35.9%, for other elements of work - 38.4% of the total time of admission of one patient. With the help of which type of diagrams can the results of the study be visually illustrated?

Картодіаграма Map Chart

Cекторна діаграма Sector chart

Радіальна діаграма Radial chart

Стовпчикова діаграма Bar chart

Лінійна діаграма Line chart

71 / 200
На рентгенограмі легень визначається ущільнення і різке зменшення верхньої частки правої легені. Середня і нижня частки правої легені різко пневмотизовані. Корінь правої легені підтягнутий до ущільненої частки. У верхньому і середньому відділах лівого легеневого поля множинні вогнищеві тіні. В прикореневій ділянці лівого легеневого поля чітко контуруються 2 поряд розміщені кільцеподібні тіні з досить товстими і нерівними стінками. Якому захворюванню відповідає вказана рентгенологічна картина? On the radiograph of the lungs, compaction and sharp reduction of the upper lobe of the right lung is determined. The middle and lower lobes of the right lung are sharply pneumatized. The root of the right lung is pulled to the compacted lobe. In the upper and there are multiple focal shadows in the middle parts of the left lung field. In the basal area of the left lung field, 2 ring-shaped shadows with rather thick and uneven walls are clearly contoured next to each other. What disease does the indicated X-ray picture correspond to?

Фіброзно-кавернозний туберкульоз легень Fibrous-cavernous pulmonary tuberculosis

Рак Пенкоста Pencost Cancer

Периферичний рак Peripheral cancer

Ателектаз верхньої частки правої легені Atelectasis of the upper lobe of the right lung

Пневмонія з абсцедуванням Pneumonia with abscess

72 / 200
Хлопчик 11-ти років за останній рік п’ять разів хворів на гострі респіраторні захворювання. До якої групи здоров’я повинен його віднести сімейний лікар? An 11-year-old boy has had acute respiratory diseases five times in the last year. To which health group should the family doctor classify him?

Перша група First group

Третя група Third group

П’ята група Fifth group

Четверта група Fourth group

Друга група Second group

73 / 200
Хвора віком 50-ти років скаржиться на біль у правій пахвинній ділянці. При дослідженні у положенні стоячи виявлено, що нижче правої шкірної пахвинної складки наявне випинання розмірами 3х3 см, яке в положенні лежачи вправляється під пахвинну зв’язку, по зовнішньому краю визначається пульсація стегнової артерії. Який найбільш імовірний діагноз? A 50-year-old patient complains of pain in the right inguinal region. During the examination in the standing position, it was found that there is a protrusion measuring 3x3 cm below the right inguinal skin fold, which is performed under the inguinal ligament in the supine position, the pulsation of the femoral artery is determined along the outer edge. What is the most likely diagnosis?

Метастаз злоякісної пухлини Malignant tumor metastasis

Стегнова грижа Femoral hernia

Пахвинний лімфаденіт Inguinal lymphadenitis

Доброякісна пухлина Benign tumor

Пахвинна грижа Inguinal hernia

74 / 200
Чоловік 59-ти років скаржиться на біль у ділянці серця, кашель, підвищення температури до 38oC. Переніс інфаркт міокарда 3 тижні тому. Об’єктивно: Ps- 86/хв, ритмічний, АТ110/70 мм рт.ст. Аускультативно: шум тертя перикарду, вологі хрипи під лопаткою. Рентгенологічно патології немає. У крові: лейк.- 10•109 /л, ШОЕ- 35 мм/год. ЕКГ - динаміки немає. Призначення препаратів якої фармакологічної групи буде найбільш обґрунтованим? A 59-year-old man complains of heart pain, cough, temperature rise to 38oC. He had a myocardial infarction 3 weeks ago. Objectively: Ps-86 /min, rhythmic, blood pressure 110/70 mm Hg. Auscultation: pericardial friction noise, moist rales under the scapula. Radiologically, there is no pathology. In the blood: leuk. - 10•109 /l, ESR - 35 mm/h. ECG - dynamics no. The prescription of drugs of which pharmacological group will be the most justified?

Глюкокортикоїди Glucocorticoids

Нітрати і нітрити Nitrates and nitrites

Фібринолітики Fibrinolytics

Антибіотики Antibiotics

Прямі антикоагулянти Direct anticoagulants

75 / 200
У хворого, що страждає на цукровий діабет, раптово опустилася верхня повіка правого ока. Лікар виявив розширення правої зіниці і розбіжну косоокість справа. Інших змін в неврологічному статусі не відмічено. Яке захворювання виникло у хворого? In a patient suffering from diabetes, the upper eyelid of the right eye suddenly drooped. The doctor found dilation of the right pupil and divergent strabismus on the right. No other changes in the neurological status were noted What disease did the patient have?

Вторинний менінгіт Secondary meningitis

Діабетична поліневропатія Diabetic polyneuropathy

Невропатія правого окорухового нерва Neuropathy of the right oculomotor nerve

Пухлина головного мозку Brain tumor

Гостре порушення мозкового кровообігу Acute cerebrovascular accident

76 / 200
Жінка 22-х років скаржиться на нудоту, блювання 1 раз на добу, сонливість, затримку менструації на 2 місяця. При бімануальному дослідженні: матка збільшена до розмірів жіночого кулака, пом’якшена, особливо в ділянці перешийка, безболісна. Придатки не пальпуються. Виділення слизові, молочного кольору. Який найбільш імовірний діагноз? A 22-year-old woman complains of nausea, vomiting 1 time a day, drowsiness, delay of menstruation for 2 months. During bimanual examination: the uterus is enlarged to the size of a woman's fist , softened, especially in the area of the isthmus, painless. The appendages are not palpable. The discharge is mucous, milky in color. What is the most likely diagnosis?

Ендометріоз матки Uterine endometriosis

Порушення менструального циклу Menstrual cycle disorder

Міома матки Uterine myoma

Позаматкова вагітність Ectopic pregnancy

Вагітність 8 тижнів Pregnancy 8 weeks

77 / 200
Хворий 17-ти років звернувся до терапевта зі скаргами на погане самопочуття, озноби, нежить, біль у м’язах та суглобах, нудоту та пронос. Просить виписати йому побільше знеболюючих та заспокійливих (трамадол або солпадеїн, котрі краще допомагають, та діазепам). Слизова зіву блідо-рожева, чиста. В легенях - везикулярне дихання. Тахікардія. Зіниці розширені, реакція на світло в’яла. На шкірі передпліч -сліди ін’єкцій. При обстеженні тримається розв’язно, роздратовано, грубий, брехливий. Встановіть діагноз: A 17-year-old patient turned to a therapist complaining of feeling unwell, chills, runny nose, muscle and joint pain, nausea and diarrhea. He asks to be prescribed more painkillers and sedatives (tramadol or solpadein, which help better, and diazepam). The mucous membrane of the pharynx is pale pink, clean. In the lungs - vesicular breathing. Tachycardia. Pupils are dilated, reaction to light is weak. On the skin of the forearm - traces of etc. ections. During the examination, he behaves loosely, irritated, rude, lying. Establish a diagnosis:

Гостре респіраторне захворювання Acute respiratory disease

Опійна наркоманія Opium addiction

Залежність від знеболюючих Addiction to painkillers

Харчова токсикоінфекція Food poisoning

Токсикоманія при зловживанні транквілізаторів Drug addiction when abusing tranquilizers

78 / 200
Потерпілий в автомобільній катастрофі доставлений в клініку нейрохірургії, оскільки були наявними ознаки черепно-мозкової травми. Під час огляду виявлені симптоми вогнищевого ушкодження мозку, запідозрено його забій. Який метод дослідження слід застосувати в даному випадку? The victim of a car accident was taken to the neurosurgery clinic, as there were signs of a brain injury. During the examination, symptoms of focal brain damage were detected, a bruise is suspected. What method research should be applied in this case?

Рентгенографія кісток черепа X-ray of skull bones

Комп’ютерна томографія Computed tomography

Обстеження очного дна Fundus examination

Ультразвукове дослідження судин головного мозку Ultrasound examination of cerebral vessels

Ангіографія судин головного мозку Angiography of cerebral vessels

79 / 200
-ти років скаржиться на підвищення температура тіла до 37,4o 79. C впродовж останніх 2-х місяців після перенесеної ГРВІ. Об’єктивно: худа, щитоподібна залоза дифузно збільшена (II ступінь), щільна при пальпації; екзофтальм, тахікардія. Який патологічний синдром має місце у хворої? year-old complains of an increase in body temperature up to 37.4o 79.C during the last 2 months after a SARS. Objectively: thin, thyroid gland diffusely enlarged (II degree), dense on palpation; exophthalmos, tachycardia. What pathological syndrome does the patient have?

Гіперпаратиреоз Hyperparathyroidism

Тиреотоксикоз Thyrotoxicosis

Гіпотиреоз Hypothyroidism

Тимомегалія Thymomegaly

Гіпопаратиреоз Hypoparathyroidism

80 / 200
Пацієнт 53-х років 5 років знаходиться під спостереженням дільничого лікаря з приводу підвищеного артеріального тиску. Протягом останнього місяця артеріальний тиск не знижувався менш ніж 160/110 мм рт.ст. Ps- 60/хв. На ЕКГ: ознаки гіпертрофії лівого шлуночка серця. Яку з груп гіпотензивних засобів слід призначити хворому як патогенетично обґрунтовану? A 53-year-old patient has been under the supervision of a district doctor for 5 years due to high blood pressure. During the last month, the blood pressure did not decrease below 160/110 mm Hg. Art. Ps- 60/min. On the ECG: signs of hypertrophy of the left ventricle of the heart. Which of the groups of hypotensive agents should be prescribed to the patient as pathogenetically justified?

Препарати групи клофеліну Clofelin group drugs

Препарати з групи раувольфії Rauwolfia drugs

Дигідроперидини пролонгованої дії Long-acting dihydroperidines

Антагоністи кальцію з підгрупи верапамілу Calcium antagonists from the verapamil subgroup

Блокатори β-адренорецепторів Blockers of β-adrenergic receptors

81 / 200
Хвора 26-ти років страждає на ревматизм з 15-ти років. Перенесла 2 ревматичні напади 4 роки тому. Протягом останніх 6-ти місяців виникали пароксизми миготливої аритмії 1 раз у 2-3 місяця. Який варіант антиаритмічної терапії або тактики слід запропонувати? A 26-year-old patient has been suffering from rheumatism since the age of 15. She suffered 2 rheumatic attacks 4 years ago. During the last 6 months, paroxysms of atrial fibrillation 1 once every 2-3 months. What option of antiarrhythmic therapy or tactics should be offered?

Профілактичний прийом кордарону Prophylactic administration of cordarone

Призначення гепарину Prescription of heparin

Негайна госпіталізація Immediate hospitalization

Дефібриляція Defibrillation

Прийом лідокаїну Lidocaine intake

82 / 200
У доношеного хлопчика на 2-гу добу життя виникла помірна жовтяниця шкіри та слизових оболонок. Загальний стан дитини не порушений. У крові: непряма гіпербілірубінемія -120 мкмоль/л. Група крові дитини A(II) Rh(+), група крові матері B(III) Rh (+). Якою повинна бути тактика лікаря? On the 2nd day of life, a full-term boy developed moderate jaundice of the skin and mucous membranes. The general condition of the child is not disturbed. Blood: indirect hyperbilirubinemia -120 μmol/l The child's blood group is A(II) Rh(+), the mother's blood group is B(III) Rh (+). What should be the doctor's tactics?

Утриматись від медикаментозної терапії Abstain from drug therapy

Призначити холекінетики Assign cholekinetics

Провести замінне переливання крові Perform replacement blood transfusion

Призначити ентеросорбенти Prescribe enterosorbents

Призначити преднізолон Prescribe Prednisone

83 / 200
При визначенні груп крові виявилась позитивною реакція ізогемаглютинації із стандартними сироватками A(II) і В(Ш) груп і негативною - із 0(1) і AB(IV) груп. Про що свідчить даний результат? When determining blood groups, the isohemagglutination reaction with standard sera of A(II) and B(Ш) groups was positive and negative - with 0(1) and AB(IV ) groups. What does this result indicate?

Друга група крові Second blood type

Непридатність стандартних сироваток Unsuitability of standard sera

Перша група крові First blood group

Четверта група крові Fourth blood type

Третя група крові Third blood group

84 / 200
При проведенні комплексу реакцій, необхідних для гемотрансфузії, хворому з виразковою хворобою шлунка з групою крові A(II) Rh+ встановлено: кров в ампулі групи A(II) Rh+ (при повторній перевірці). При реакції на індивідуальну сумісність за резус-фактором - аглютинація є. Яка подальша тактика? When conducting a set of reactions necessary for hemotransfusion, a patient with peptic ulcer disease with blood group A(II) Rh+ was found to have: blood in an ampoule of group A(II) Rh+ (when re-checking). When reacting to individual compatibility according to the Rhesus factor - there is agglutination. What is the next tactic?

Перелити хворому кров А(П) Rh+ Transfuse the patient with A(P) Rh+ blood

!ндивідуальний підбір крові на станції переливання крові !individual selection of blood at the blood transfusion station

Перелити хворому вказану кров з профілактикою гемотрансфузійного шоку Transfuse the patient with the specified blood to prevent hemotransfusion shock

Повністю відмовитися від проведення гемотрансфузії Completely refuse blood transfusion

Повторно провести індивідуальну пробу на резус-сумісність Re-conduct an individual test for Rh compatibility

85 / 200
У хворої 53-х років діагностований гострий абсцес легені. З анамнезу виявлено, що 2 тижні тому вона була виписана у задовільному стані із терапевтичного стаціонару, де знаходилась з приводу гострої нижньодольової пневмонії. Назвіть найбільш імовірний шлях виникнення абсцесу легені у хворої: A 53-year-old patient was diagnosed with an acute lung abscess. From the anamnesis, it was found that 2 weeks ago she was discharged in a satisfactory condition from the therapeutic hospital, where she was due to of acute lower lobe pneumonia. Name the most likely way of occurrence of a lung abscess in a patient:

Бронхіальна обструкція Bronchial obstruction

Лімфогенний шлях інфікування Lymphogenic route of infection

Травматичний шлях інфікування Traumatic route of infection

Емболічний шлях інфікування Embolic route of infection

Гематогенний шлях інфікування Hematogenous route of infection

86 / 200
звернувся хворий з карбункулом верхньої губи. Температура тіла 39o 86. C. Виражений набряк верхньої губи, повік. Як повинен вчинити хірург з цим хворим? a patient came in with a carbuncle of the upper lip. Body temperature 39o 86. C. Pronounced swelling of the upper lip, eyelid. How should the surgeon treat this patient?

Амбулаторно призначити курс антибіотикотерапії Prescribe a course of antibiotic therapy on an outpatient basis

Призначити фізіотерапевтичні процедури і зігріваючі компреси Prescribe physiotherapeutic procedures and warming compresses

Розкрити карбункул і призначити антибіотики Open the carbuncle and prescribe antibiotics

Госпіталізувати до хірургічного відділення Hospitalize to the surgical department

Розкрити карбункул, лікувати амбулаторно Open carbuncle, treat on an outpatient basis

87 / 200
У хворого 38-ми років після автокатастрофи діагностовано перелом кісток тазу. Протягом 24-х годин турбує часте, болюче, малими порціями, з домішкою крові сечовипускання. На висхідній цистограмі, яку виконано в зв’язку з підозрою на розрив сечового міхура, - сечовий міхур у вигляді боксерської груші, променеподібне затікання контрастної речовини. Яку лікувальну тактику слід застосувати? A 38-year-old patient was diagnosed with a fracture of the pelvic bones after a car accident. Within 24 hours, frequent, painful, small portions, with blood in the urine are bothering him. On the ascending cystogram, which was performed in connection with the suspicion of rupture of the urinary bladder - the bladder in the form of a punching bag, radio-like infiltration of the contrast agent. What treatment tactics should be used?

Термінове хірургічне втручання Urgent surgery

Капілярна пункція сечового міхура Capillary puncture of the bladder

Холод на надлобкову ділянку, спостереження Cold in suprapubic area, observation

Встановлення постійного уретрального катетера Indwelling urethral catheter

Антибактеріальна, гемостатична терапія Antibacterial, hemostatic therapy

88 / 200
Хворий 22-х років на другий день хвороби скаржиться на високу температуру, головний біль у ділянці чола, надбрівних дуг, при русі очними яблуками, у м’язах і суглобах. Об’єктивно: температура - 390 С. Обличчя гіперемоване, склери ін’єковані. Слизова оболонка м’якого піднебіння, задньої стінки глотки яскраво гіперемована з крапковими крововиливами. Які зміни в гемограмі типові для цього захворювання? On the second day of the illness, a 22-year-old patient complains of a high temperature, headache in the forehead, browbones, when moving the eyeballs, in the muscles and joints. Objectively: the temperature is 390 C. The face is hyperemic, the sclera is injected. The mucous membrane of the soft palate, the back wall of the pharynx is clearly hyperemic with punctate hemorrhages. What changes in the hemogram are typical for this disease?

Лейкоцитоз Leukocytosis

Лейкопенія Leukopenia

Прискорена ШОЕ Accelerated ESR

Нейтрофільоз Neutrophilosis

Анемія Anemia

89 / 200
У хлопчика 14-ти років із загостренням вторинного обструктивного пієлонефриту із сечі виділена синьогнійна паличка в титрі 1000000 мікробних тіл на 1 мл. Який антибактеріальний препарат найбільш доцільно призначити в даному випадку? A 14-year-old boy with an exacerbation of secondary obstructive pyelonephritis was isolated from the urine of Pseudomonas aeruginosa with a titer of 1,000,000 microbial bodies per 1 ml. Which antibacterial drug is the most appropriate to prescribe in this case case?

Азитроміцин Azithromycin

Левоміцетин Levomycetin

Ампіцилін Ampicillin

Цефазолін Cefazolin

Ципрофлоксацин Ciprofloxacin

90 / 200
У хворого 45-ти років після тупої травми грудної клітки з переломом груднини виникли слабкість, гіпотонія, ціаноз верхньої половини тулуба, набухання вен шиї. При плевральній пункції вміст відсутній. Ps- 120/хв., ритмічний, слабкого наповнення. Який діагноз найбільш імовірний? A 45-year-old patient developed weakness, hypotonia, cyanosis of the upper half of the body, swelling of the neck veins after a blunt trauma to the chest with a fracture of the sternum. Pleural puncture showed no contents . Ps- 120/min., rhythmic, weak filling. What diagnosis is most probable?

Тампонада серця Cardiac tamponade

Згорнений гемоперикард Collapsed hemopericardium

Тромбоемболія легеневої артерії Thromboembolism of the pulmonary artery

Гострий інфаркт міокарда Acute myocardial infarction

Струс серця Heartshock

91 / 200
У хворого опік полум’ям 2А, 3А, Б ступеня обох рук, грудей, живота. Загальна площа опіку приблизно 35 % поверхні тіла, з них глибоких - 28% поверхні тіла. Опік отримав 4 тижні тому. Загальний стан хворого важкий, тем- пература тіла 38,10 C, Ps- 92/хв., аритмічний, АТ- 125/70 мм рт.ст. Який найбільш імовірний діагноз? The patient has 2A, 3A, B degree flame burns on both arms, chest, abdomen. The total area of the burn is approximately 35% of the body surface, of which 28 are deep % of the body surface. He received a burn 4 weeks ago. The patient's general condition is serious, body temperature 38.10 C, Ps- 92/min., arrhythmic, BP- 125/70 mm Hg. What is the most likely diagnosis?

Реконвалесценція Convalescence

Хронічний опіковий шок Chronic burn shock

Опіковий шок Burn shock

Гостра опікова токсемія Acute burn toxemia

Опікова сєптикопіємія Burn septicopyemia

92 / 200
Хвора 32-х років висловлює скарги на знижений настрій. Чує 'голоси' сусідів, які їй загрожують, коментують її дії. Вважає, що вони стежать за нею через стіни, на вулиці, у магазині. Визначте синдром: A 32-year-old patient complains of depressed mood. She hears the 'voices' of her neighbors who threaten her, comment on her actions. She believes that they are watching her because walls, on the street, in a store. Identify the syndrome:

Парафренний Paraphrenic

Параноїдний Paranoid

Депресивний Depressed

Паранояльний Paranoid

Галюциноз Hallucinosis

93 / 200
У хлопчика 14-ти років на тлі хронічного тонзиліту та гаймориту появились відчуття перебоїв у ділянці серця і додаткових пульсових ударів. ЧСС-83/хв. На ЕКГ: після кожних двох синусових скорочень регулярно виникають імпульси, в яких відсутній зубець P, QRS поширений більше 0,11 с, різко деформований, дискордантний зубець T, після чого реєструється повна компенсаторна пауза. Вкажіть характер порушень ритму: A 14-year-old boy, against the background of chronic tonsillitis and sinusitis, felt interruptions in the heart area and additional pulse beats. HR-83/min. On the ECG: after every two sinus contractions, pulses regularly occur in which there is no P wave, QRS is extended for more than 0.11 s, sharply deformed, discordant T wave, after which a complete compensatory pause is recorded. Specify the nature of rhythm disturbances:

Часткова AV-блокада Partial AV block

Блокада лівої ніжки пучка Гіса Blockade of the left leg of the bundle of His

Повна AV-блокада Full AV block

Екстрасистолія за типом бігемінії Extrasystole by type of bigeminy

Екстрасистолія за типом тригемінії Extrasystole according to the type of trigeminy

94 / 200
У дитини 10-ти років з неревматичним кардитом періодично виникають напади, які проявляються відчуттям болю в ділянці серця, задишкою, блідістю, підвищенням артеріального тиску, різким збільшенням ЧСС до 180/хв. Який з медикаментозних засобів є найбільш ефективним у лікуванні? A 10-year-old child with non-rheumatic carditis periodically has attacks, which are manifested by a feeling of pain in the heart area, shortness of breath, pallor, an increase in blood pressure, a sharp increase in heart rate to 180/min. Which medication is the most effective in treatment?

Новокаїнамід Novocaineamide

Аймалін Aimalin

Верапаміл Verapamil

Лідокаїн Lidocaine

Обзідан Obzidan

95 / 200
У хворого 38-ми років внаслідок отриманого удару тупим предметом по лівій половині грудної клітки виявлено перелом Х ребра зліва зі зміщенням відламків, пристінковий пневмоторакс. Відзначає болі у лівому підребер’ї. Об’єктивно: блідий, АТ- 80/40 мм рт.ст., Ps- 138/хв., слабких наповнення і напруження. На УЗД виявлено рідину в лівій половині живота. Виявлено розрив селезінки. Яку лікувальну тактику слід обрати? A 38-year-old patient, as a result of a blow to the left half of the chest with a blunt object, was diagnosed with a fracture of the X rib on the left with displacement of the fragments, a parietal pneumothorax. He notes pain in the left hypochondrium Objectively: pale, blood pressure - 80/40 mm Hg, Ps - 138/min, weak filling and straining. Fluid was detected in the left half of the abdomen on ultrasound. A rupture of the spleen was detected. What treatment tactics should be chosen ?

Провести протишокові заходи і після підвищення артеріального тиску зробити лапаротомію Carry out anti-shock measures and, after raising the blood pressure, perform a laparotomy

Дренувати ліву плевральну порожнину і зробити лапаротомію Drain the left pleural cavity and perform a laparotomy

Зробити негайно лапаротомію і спирт-новокаїнову блокаду Х ребра Perform immediate laparotomy and alcohol-novocaine blockade of X rib

Негайно виконати верхньо-серединну лапаротомію і потім дренувати ліву плевральну порожнину Immediately perform an upper-middle laparotomy and then drain the left pleural cavity

Зробити лівобічну торакотомію, а потім одразу ж лапаротомію Perform left-sided thoracotomy and then immediately laparotomy

96 / 200
У хворої 26-ти років, оперованої з приводу дифузного токсичного зобу III ступеня, тиреотоксикозу середньої важкості, на 2 добу після операції з’явилися судоми кистей, стоп та обличчя. Симптоми Хвостека, Трусо позитивні. Хвора скаржиться на болі в ділянці серця. На ЕКГ - подовження інтервалу Q-T Яке ускладнення виникло у хворої? A 26-year-old patient, operated on for diffuse toxic goiter III degree, thyrotoxicosis of moderate severity, on the 2nd day after the operation, spasms of the hands, feet and face. Khvostek's and Truso's symptoms are positive. The patient complains of pain in the area of the heart. On the ECG - prolongation of the QT interval What complication did the patient have?

Парез гортанних нервів Paresis of the laryngeal nerves

Тиреотоксичний криз Thyrotoxic crisis

Гіперпаратиреоз Hyperparathyroidism

Гіпопаратиреоз Hypoparathyroidism

Тиреотоксична міокардіодистрофія Thyrotoxic myocardial dystrophy

97 / 200
Хворий 36-ти років поступив із скаргами на кашель з виділенням гнійного харкотиння до 150 мл на добу, кровохаркання, періодичне підвищення температури до 37,8oC, знижений апетит, загальну слабкість. Хворіє протягом 10-ти років, загострення захворювання навесні і восени. Об’єктивно: блідість, легкий акроціаноз, потовщення нігтьових фаланг пальців. Аускультативно: вологі хрипи під лівою лопаткою. На Roграмах: розширення кореня легені, тяжистість і шпаристість легеневого малюнка зліва. Який найбільш вірогідний метод для уточнення діагнозу? A 36-year-old patient was admitted with complaints of a cough with purulent sputum up to 150 ml per day, hemoptysis, a periodic increase in temperature up to 37.8oC, decreased appetite, general weakness. He has been sick for 10 years, exacerbation of the disease in the spring and autumn. Objectively: paleness, slight acrocyanosis, thickening of the nail phalanges of the fingers. Auscultatively: moist rales under the left shoulder blade. On Rograms: expansion of the lung root, heaviness and spasminess of the lung pattern on the left. What is the most likely method for clarifying the diagnosis?

Торакоскопія Thoracoscopy

Томографія Tomography

Бронхографія Bronchography

Рентгеноскопія X-ray

Бронхоскопія Bronchoscopy

98 / 200
У 48-річної жінки ІХС на протязі півроку з прогресуючими нападами стенокардії. Після проведеної коронаровентрікулографії виявлено: стеноз > 70% передньої міжшлуночкової артерії (ПМША) в середній третині на протязі 0,7 см. В інших артеріях - пристінкові атеросклеротичні зміни, гемодинамічно незначимі. Хворій показано: A 48-year-old woman had coronary heart disease for six months with progressive angina attacks. Coronary ventriculography revealed: stenosis > 70% of the anterior interventricular artery (AVA) in the middle third of for 0.7 cm. In other arteries, wall atherosclerotic changes are hemodynamically insignificant. The patient is shown:

Балонна ангіопластика ПМША Balloon angioplasty of PMS

Медикаментозна терапія Drug therapy

Операція МКШ до ПМША Operation MKS to PMSHA

Операція АКШ 2-3 артерій CABG operation of 2-3 arteries

Операція АКШ однієї артерії Single artery CABG operation

99 / 200
При вивченні розкладу занять учнів 3-го класу встановлено: кількість уроків протягом тижня - 30; у вівторок перший урок українська мова, 2-й - музика, 3 і 4 - фізичне виховання, усього 5 уроків. Яке провідне місце в розкладі уроків? When studying the class schedule of 3rd grade students, it was established that: the number of lessons during the week is 30; on Tuesday, the first lesson is the Ukrainian language, the 2nd is music, 3 and 4 - physical education, only 5 lessons. What is the leading place in the lesson schedule?

Кількість уроків фізкультури Number of physical education lessons

Кількість уроків протягом дня Number of lessons during the day

Місце уроку української мови в розкладі Place of the Ukrainian language lesson in the schedule

Кількість уроків протягом тижня Number of lessons during the week

Місце уроку музики в розкладі Place of the music lesson in the schedule

100 / 200
На підприємстві в процесі виробництва утворюються особливо токсичні неутилізовані промислові відходи. Запропонуйте метод утилізації та знешкодження: In the production process, particularly toxic unutilized industrial waste is generated at the enterprise. Suggest a method of utilization and disposal:

Захоронення в котлованах полігонів в контейнерній тарі Burial in landfill pits in a container

Захоронення в котлованах полігонів з ізоляцією дна і стінок ущільненим шаром глини Burial in landfill pits with insulation of the bottom and walls with a compacted layer of clay

Використання як сировини для повторної переробки Use as raw material for reprocessing

Термічна обробка Thermal treatment

Біотермічна переробка на удосконалених звалищах Biothermal processing at advanced landfills

101 / 200
У дитини 1 місяця зригування, що виникають щоразу, як тільки після годування грудьми малюка вкладають у ліжечко. Під час огляду педіатр відхилень у стані дитини не виявив. Виникнення зригувань лікар пояснив особливостями травної системи в даному віці та дав поради по догляду за дитиною. В чому полягають ці поради? A 1-month-old child has vomiting, which occurs every time the baby is placed in a crib after breastfeeding. During the examination, the pediatrician did not find any abnormalities in the child's condition. Vomiting occurs The doctor explained the peculiarities of the digestive system at this age and gave advice on caring for the child. What are these tips?

Вертикальне положення дитини зразу після годування Vertical position of the child immediately after feeding

Теплові процедури на живіт Heat procedures on the stomach

Укладання дитини на живіт після годування Putting baby on stomach after feeding

Дача соски зразу після годування грудьми Giving a nipple immediately after breastfeeding

Масаж живота Abdominal massage

102 / 200
До дільничного педіатра звернулася мати 4,5 місячної дівчинки за порадою стосовно частоти проведення дитині гігієнічних ванн. Яка частота проведення гігієнічних ванн є найбільш прийнятною у цьому віці? The mother of a 4.5-month-old girl turned to the district pediatrician for advice on the frequency of hygienic baths for the child. What frequency of hygienic baths is the most acceptable at this age?

1 раз на 2 дні 1 time in 2 days

1 раз на день 1 time per day

1 раз на 10 днів 1 time in 10 days

1 раз на тиждень Once a week

1 раз на 3 дні 1 time in 3 days

103 / 200
Хвора 55-ти років скаржиться на ниючий біль у епігастральній ділянці, нудоту, печію. Зазначені прояви з’явилися після лікування індометацином. Об’єктивно: живіт м’який, болючий в епігастральній ділянці. Печінка, селезінка не збільшені. Відділи кишечнику безболісні. Що треба здійснити в даному випадку в першу чергу? A 55-year-old patient complains of aching pain in the epigastric region, nausea, heartburn. These symptoms appeared after treatment with indomethacin. Objectively: the stomach m' which is painful in the epigastric area. The liver and spleen are not enlarged. The intestinal sections are painless. What should be done in this case first of all?

Відмінити індометацин Cancel indomethacin

Призначити голод на 2 дні Set hunger for 2 days

Призначити антацидні засоби Prescribe antacids

Призначити препарати, які впливають на Helicobacter pylori Prescribe drugs that affect Helicobacter pylori

Промити шлунок Wash the stomach

104 / 200
Жінка 27-ми років, що активно живе статевим життям, скаржиться на наявність численних везикул на правій статевій губі, свербіж та печію. Висипання періодично з’являються перед менструацією і зникають через 8-10 днів. Який найбільш імовірний діагноз? A 27-year-old woman with an active sexual life complains of the presence of numerous vesicles on the right labia, itching and heartburn. Rashes periodically appear before menstruation and disappear after 8-10 days. What is the most likely diagnosis?

Первинний сифіліс Primary syphilis

Генітальний кондиломатоз Genital condylomatosis

Бартолініт Bartholinitis

Вірус простого герпесу Herpes simplex virus

Цитомегаловірусна інфекція Cytomegalovirus infection

105 / 200
У хворого 33-х років після введення пеніциліну з’явилася уртикарна висипка на шкірі тулуба і кінцівок, загальна слабість. Під час транспортування хворого машиною швидкої допомоги до спеціалізованого відділення у нього виник напад ядухи. Об’єктивно: ЧД28/хв., Ps- 94/хв., АТ- 100/60 мм рт.ст. Який спосіб вентиляції легень найбільш доцільний у даному випадку? A 33-year-old patient developed an urticarial rash on the skin of the trunk and limbs, general weakness after the administration of penicillin. During the transportation of the patient by ambulance to a specialized department he had an attack of dyspnoea. Objectively: BH 28/min., Ps- 94/min., BP- 100/60 mm Hg. What method of lung ventilation is most appropriate in this case?

Застосування мішка Амбу Application of Ambu Bag

Інтубація трахеї Tracheal intubation

Застосування кисневої маски Application of oxygen mask

Застосування ларингеальної маски Application of laryngeal mask

Застосування повітроводу Conductor Application

106 / 200
У хворого 38-ми років раптово з’явився біль у лівій половині грудної клітки, ядуха. Об’єктивно: стан середньої важкості, Ps- 100/хв., АТ- 90/60 мм рт.ст., дихання злiва не вислуховується. При рентгенографії грудної клітки - колапс лівої лєгєні до 1/2. Яке лікування потрібно призначити хворому? A 38-year-old patient suddenly developed pain in the left half of the chest, a dull ache. Objectively: the condition is of medium severity, Ps- 100/min. , blood pressure - 90/60 mmHg, breathing from the left side is not heard. Chest x-ray shows the collapse of the left lung to 1/2. What treatment should be prescribed to the patient?

Активне дренування плевральної порожнини Active drainage of the pleural cavity

Оперативне лікування Operative treatment

Покій, розсмоктуюча терапія Rest, absorption therapy

Плевральні пункції Pleural punctures

Пасивне дренування плевральної порожнини Passive drainage of the pleural cavity

107 / 200
Хвора 35-ти років звернулася до гінекологічного стаціонару зі скаргами на періодичні болі в нижніх відділах живота, що посилюються під час менструації, темно-коричневі мажучі виділення зі статевих шляхів. При бімануальному дослідженні: тіло матки трохи збільшено, придатки не визначаються, при дзеркальному дослідженні шийки матки виявляються синюшні 'глазки'. Який з наступних діагнозів найбільш імовірний? A 35-year-old patient turned to a gynecological hospital with complaints of periodic pain in the lower abdomen, which worsens during menstruation, dark brown smearing discharge from the genital tract . During a bimanual examination: the body of the uterus is slightly enlarged, the appendages are not defined, during a speculum examination of the cervix, blue eyes are revealed. Which of the following diagnoses is the most probable?

Фіброїд шийки матки Cervical fibroid

Поліп шийки матки Cervical polyp

Рак шийки матки Cervical cancer

Ерозія шийки матки Cervical erosion

Ендометріоз шийки матки Cervical endometriosis

108 / 200
Хвора 51-го року скаржиться на постійні кров’яні виділення із статевих шляхів мажучого характеру протягом останніх 3-х місяців, контактні кровотечі. При бімануальному обстеженні: шийка матки збільшена, обмежена в рухомості, щільна на дотик. У дзеркалах: кратероподібна виразка по центру. Проба Хробака позитивна. Який з наступних діагнозів найбільш імовірний? A 51-year-old patient complains of constant bleeding from the genital tract of a smearing nature during the last 3 months, contact bleeding. During bimanual examination: cervix enlarged, limited in mobility, dense to the touch. In the mirrors: a crater-shaped ulcer in the center. The Worm test is positive. Which of the following diagnoses is most likely?

Рак шийки матки Cervical cancer

Поліп шийки матки Cervical polyp

Ерозія шийки матки Cervical erosion

Лейкоплакія шийки матки Leukoplakia of the cervix

Шийкова вагітність Cervical pregnancy

109 / 200
До гінекологічного стаціонару звернулася жінка 36-ти років зі скаргами на значну кровотечу зі статевих шляхів та затримку місячних на місяць. При бімануальному дослідженні: шийка матки бочкоподібної форми, м’якої консистенції. Матка звичайних розмірів, дещо розм’якшена. Придатки без особливостей з обох сторін. При дзеркальному дослідженні: шийка матки синюшна, збільшена у розмірах, зовнішнє вічко розкрите до 0,5 см. Дослідження сечі на ХГ - позитивне. Який найбільш імовірний діагноз? A 36-year-old woman came to the gynecological hospital with complaints of significant bleeding from the genital tract and a delay of menstruation for a month. On bimanual examination: the cervix is barrel-shaped, m 'what consistency. Uterus of normal size, somewhat softened. Appendages without features on both sides. On endoscopy: the cervix is bluish, enlarged, the external eye is open up to 0.5 cm. Urine test for HCG is positive. Which is the most probable diagnosis?

Позаматкова вагітність Ectopic pregnancy

Шийкова вагітність Cervical pregnancy

Маткова вагітність Uterine pregnancy

Аборт у ходу Abort in progress

Загроза переривання вагітності Threat of abortion

110 / 200
У хворої 64-х років діагностований рак шлунка, вона готується до радикальної операції. Має місце супутня патологія: посттромбофлебітичний синдром, набрякло-больова форма. В анамнезі - тромбоемболія легеневої артерії 3 роки тому. Вкажіть найбільш ефективний метод профілактики розвитку у хворої повторної ТЕЛА у післяопераційному періоді після радикальної операції на шлунку: A 64-year-old patient has been diagnosed with stomach cancer, she is preparing for a radical operation. There is a concomitant pathology: post-thrombophlebitic syndrome, edematous and painful form. She has a history of thromboembolism of the pulmonary artery 3 years ago. Specify the most effective method of preventing the development of recurrent PE in a patient in the postoperative period after a radical operation on the stomach:

Імплантація кава-фільтру в передопераційному періоді Cava filter implantation in the preoperative period

Призначення непрямого антикоагулянту в перед- та післяопераційному періоді Prescription of indirect anticoagulant in the pre- and postoperative period

Накладання цинк-желатинової пов’язки Унна у передопераційному періоді Applying Unn's zinc-gelatin bandage in the preoperative period

Гепаринотерапія у післяопераційному періоді Heparin therapy in the postoperative period

Застосування еластичного бинтування кінцівок у післяопераційному періоді Application of elastic bandaging of limbs in the postoperative period

111 / 200
Хвора 47-ми років хворіє на виразковий коліт 8 років, лікувалась глюкокортикоїдами. Скаржиться на переймоподібний біль у навколопупковій та в лівій здухвинній ділянках, який за останні 2 тижні значно підсилився, пронос із слизом та кров’ю 4-6 разів на добу, підвищення температури тіла до 38 — 39oC, головний біль та біль у колінних суглобах. Об’єктивно: стан хворої середньої важкості, Ps- 108/хв., АТ-90/60 мм рт.ст., серце і легені без особливостей, язик вологий, тонус м’язів живота значно знижений, перистальтичні шуми відсутні. Яке ускладнення розвинулось у хворої? A 47-year-old patient has been suffering from ulcerative colitis for 8 years and has been treated with glucocorticoids. She complains of cramp-like pain in the periumbilical and left iliac regions, which in the last 2 weeks has significantly increased, diarrhea with mucus and blood 4-6 times a day, increase in body temperature to 38-39oC, headache and pain in the knee joints. Objectively: the patient's condition is of moderate severity, Ps- 108/min., BP 90/60 mm Hg, the heart and lungs are normal, the tongue is wet, the tone of the abdominal muscles is significantly reduced, there are no peristaltic sounds. What complication developed in the patient?

Стриктура товстої кишки Colon stricture

Перфорація товстої кишки Perforation of the colon

Кишкова кровотеча Intestinal bleeding

Рак товстої кишки Colon cancer

Токсична дилятація товстої кишки Toxic colon dilatation

112 / 200
Хворий 64-х років звернувся до уролога зі скаргами на утруднене, млявим струменем, сечовипускання. При ректальному дослідженні виявлені зміни простати, що не дозволяють диференціювати аденому і рак. Яке дослідження треба провести для уточнення діагнозу? A 64-year-old patient turned to a urologist with complaints of difficulty urinating with a sluggish stream. During a rectal examination, changes in the prostate were detected, which do not allow differentiation of adenoma and cancer. What research should be conducted to clarify the diagnosis?

Уретроцистографія Urethrocystography

Загальний аналiз секрету простати General analysis of prostate secretion

Визначення кількості залишкової сечі Determining the amount of residual urine

Дослідження крові на простатоспецифічний антиген Blood test for prostate-specific antigen

Визначення рівня сечовини і креатиніну сироватки крові Determination of blood serum urea and creatinine

113 / 200
На авіаційному заводі проводиться обробка матеріалів з використанням оптичних квантових генераторів. Встановлено, що установка випромінює у видимому спектрі, рівні лазерного випромінювання на робочих місцях перевищують ГДР Який орган буде уражатись в першу чергу? Materials are processed using optical quantum generators at the aviation plant. It was established that the installation emits in the visible spectrum, the levels of laser radiation at workplaces exceed the GDR Which organ will be affected first of all?

Нирки Kidneys

Печінка Liver

Селезінка Spleen

Очі Eyes

Шкіра Skin

114 / 200
В поліклініку звернувся пенсіонер 72-х років. Ранком цього ж дня на дачі наступив на цвях і травмував праву стопу. Даних про попередні вакцинації відсутні. Об’єктивно: стан задовільний. Права стопа незначно набрякла, на підошві колота рана. З метою запобігання можливого розвитку правця насамперед потрібно: A 72-year-old pensioner came to the polyclinic. In the morning of the same day, he stepped on a nail in the country and injured his right foot. There are no data on previous vaccinations. Objectively: the condition is satisfactory. The right foot is slightly swollen, there is a puncture wound on the sole. In order to prevent the possible development of tetanus, first of all it is necessary:

Обробити рану мильним розчином Treat the wound with a soap solution

В/м ввести 3000 МО протиправцевої сироватки Inject 3000 IU of anti-tetanus serum

В/м ввести 1 мл правцевого анатоксину, 3000 МО протиправцевої сироватки Inject 1 ml of tetanus toxoid, 3000 IU of anti-tetanus serum

В/м ввести 0,5 мл правцевого анатоксину Inject 0.5 ml of tetanus toxoid intravenously

Призначити курс антибіотикотерапії Prescribe a course of antibiotic therapy

115 / 200
Хворий 27-ми років скаржиться на біль у правому оці, що посилюється вночі, зниження зору, світлобоязнь, сльозотеча. Тиждень тому перехворів на грип. Об’єктивно: очна щілина звужена, зіниця вузька, колір райдужної оболонки змінений, опалесценція вмісту передньої камери. Циклічна болісність. Який найбільш імовірний діагноз? A 27-year-old patient complains of pain in the right eye that worsens at night, decreased vision, photophobia, lacrimation. A week ago he got sick with the flu. Objectively: the eye slit is narrowed, the pupil is narrow, the color of the iris is changed, the content of the anterior chamber is opalescent. Cyclic pain. What is the most likely diagnosis?

Кератит Keratitis

Ірит Irit

Дакріоденіт Dacryodenitis

Іридоцикліт Iridocyclite

Кон’ юнктивіт Conjunctivitis

116 / 200
років хворіє 3-й день: температура 38 — 38,5o 116. C, незначний біль при ковтанні, збільшені передньошийні лімфовузли. При огляді: піднебінні мигдалики набряклі, їх поверхня вкрита білувато-сірими нальотами з гладенькою поверхнею, які щільно зв’язані з прилеглими тканинами. Встановлено діагноз дифтерії. Який процес лежить в основі утворення дифтеритичних нальотів? years sick on the 3rd day: temperature 38 — 38.5o 116. C, slight pain when swallowing, enlarged anterior cervical lymph nodes. On examination: palatine tonsils are swollen, their surface is covered with whitish-gray plaques with a smooth surface, which are tightly connected to the surrounding tissues. The diagnosis of diphtheria is established. What process underlies the formation of diphtheritic plaques?

Некротичний процес Necrotic process

Дистрофічний процес Dystrophic process

Катаральне запалення Catarrhal inflammation

Фібринозне запалення Fibrinous inflammation

Гнійне запалення Purulent inflammation

117 / 200
У новонародженого, що народився в результаті стрімких пологів, спостерігається парез м’язів кисті. Хапальний рефлекс відсутній, не вдається викликати долонно-ротовий рефлекс. Чутливість кисті відсутня. Який найбільш імовірний діагноз? A newborn born as a result of rapid delivery has paresis of the muscles of the hand. The grasping reflex is absent, it is not possible to trigger the hand-mouth reflex. The sensitivity of the hand is absent. What is the most likely diagnosis?

Парез Дежерін-Клюмпке Dezherin-Klumpke paresis

Синдром Горнера-Бернара Horner-Bernard Syndrome

Тотальне ураження плечового сплетення Total lesion of brachial plexus

Парез діафрагми Paresis of the diaphragm

Парез Дюшена-Ерба Duchen-Erb paresis

118 / 200
Хвора 30-ти років в психіатричному відділенні демонстративна, вередлива, балакуча, постійно привертає до себе увагу оточуючих. Скаржиться на болі у всьому тілі. Побачивши лікаря - стогне, хватається за голову, голосить, демонструє неможливість ходити, тримається за оточуючі предмети. Наодинці вільно ходить по палаті, співає, накладає макіяж. Визначте стан хворої: A 30-year-old patient in a psychiatric ward is demonstrative, cranky, talkative, constantly attracts the attention of others. She complains of pain in her whole body. When she sees the doctor, she moans, grabs his head, wails, demonstrates inability to walk, holds on to surrounding objects. Walks freely around the ward alone, sings, applies makeup. Determine the condition of the patient:

Маніакально-депресивний психоз Manic-depressive psychosis

Іпохондричний розвиток особистості Hypochondriac personality development

Шизофренія Schizophrenia

Невроз нав’язливих станів Compulsive neurosis

Істеричний невроз Hysterical neurosis

119 / 200
Хворий 30-ти років надійшов у приймальне відділення після автомобільної аварії зі скаргами на задишку, біль у лівому боці. На оглядовій рентгенограмі органів грудної клітки визначається просторе просвітлення лівого легеневого поля з відсутністю легеневого малюнка, зміщення органів середостіння вправо. Ліва легеня притиснута до лівого кореня, діафрагма зміщена на одне міжребер’я донизу, синуси контуруються чітко. Який найбільш імовірний діагноз? A 30-year-old patient was admitted to the emergency department after a car accident with complaints of shortness of breath, pain in the left side. On the X-ray examination of the chest organs, a spacious luminescence of the left pulmonary fields with no lung pattern, displacement of the mediastinal organs to the right. The left lung is pressed against the left root, the diaphragm is shifted one intercostal space down, the sinuses are clearly contoured. What is the most likely diagnosis?

Забій легені Lung contusion

Пневмоторакс Pneumothorax

Пошкодження діафрагми Diaphragm damage

Гемоторакс Hemothorax

Пневмогемоторакс Pneumothorax

120 / 200
Хворий 81-го року скаржиться на постійне виділення сєчі краплями, відчуття розпирання внизу живота. Об’єктивно: над лобком кулеподібне випинання, над яким перкуторно визначається притуплення, позитивний надлобковий поштовх. Який симптом спостерігається у хворого? An 81-year-old patient complains of a constant release of urine in drops, a feeling of distention in the lower abdomen. Objectively: a ball-shaped protrusion above the pubis, over which dullness is determined by percussion, positive suprapubic impulse. What symptom is observed in the patient?

Енурез Enuresis

Неутримання сечі Urinary incontinence

Парадоксальна ішурія Paradoxical Ischuria

Полакіурія Polakiuria

Дизурія Dysuria

121 / 200
Хворий 50-ти років звернувся зі скаргами на підвищення температури тіла до 39oC, пульсуючий біль та припухлість правої кисті. Напередодні уколов кисть риб’ячою кісткою. При огляді відмічається набряк і різкий біль у долоні та в ділянці підвищення I пальця кисті. Окрім цього відзначається набряк м’яких тканин тилу кисті, почервоніння шкіри та різкий біль при рухах пальців. Яке захворювання у пацієнта? A 50-year-old patient complained of an increase in body temperature to 39oC, throbbing pain and swelling of the right hand. The day before, he pricked the hand with a fish bone. During the examination, it is noted swelling and sharp pain in the palm and in the area of the first finger of the hand. In addition, there is swelling of the soft tissues of the back of the hand, redness of the skin and sharp pain when moving the fingers. What disease does the patient have?

Флегмона правої кисті Phlegmon of the right hand

Карбункул кисті Carbuncle of the hand

Сухожилковий панарицій I пальця кисті Tendinosus panaritium of the first finger

Стороннє тіло кисті Foreign body of the brush

Бешиха кисті Beshikha brush

122 / 200
У місті на вибірковій сукупності вивчався вплив викидів у повітря відходів металургійного виробництва на захворюваність на обструктивний бронхіт. Розрахований коефіцієнт кореляції становив +0,79. Оцініть силу і напрямок зв’язку: In the city, the influence of emissions of metallurgical waste into the air on the incidence of obstructive bronchitis was studied on a sample population. The calculated correlation coefficient was +0.79. Estimate the strength and direction of the 'link:

Прямий, сильний Direct, strong

Зворотній, сильний Reverse, strong

- -

Прямий, середній Direct, Average

Зворотній, середній Reverse, Average

123 / 200
Службовець хворіє 4 місяці, необхідно подальше лікування, хворий непрацездатний. Хто уповноважений проводити експертизу непрацездатності даного хворого? The employee has been ill for 4 months, further treatment is required, the patient is incapacitated. Who is authorized to conduct an examination of the incapacity of this patient?

Лікарсько-консультативна комісія Medical Advisory Committee

Медико-соціальна експертна комісія Medical and social expert commission

Заступник головного лікаря по експертизі непрацездатності Deputy Chief Physician for Incapacity Examination

Лікуючий лікар із завідувачем відділення Treating doctor with department head

Головний лікар лікувально-профілактичного закладу Chief physician of the medical and preventive institution

124 / 200
У чоловіка 43-х років при проведенні медичного огляду об’єктивно виявлені блідість шкіри і слизових оболонок, згладженість сосочків язика, поперечна посмугованість нігтів, тріщини у кутиках рота, тахікардія. У крові: Hb-90 г/л, анізоцитоз, пойкілоцитоз. Найбільш імовірним причинним фактором даного стану є недостатнє надходження до організму: The medical examination of a 43-year-old man objectively revealed pallor of the skin and mucous membranes, smoothing of the papillae of the tongue, transverse striations of the nails, cracks in the corners of the mouth, tachycardia. In the blood: Hb-90 g/l, anisocytosis, poikilocytosis. The most likely causal factor of this condition is insufficient supply to the body:

Міді Copper

Цинку Zinc

Заліза Iron

Селену Selena

Магнію Magnesium

125 / 200
Хвора 29-ти років скаржиться на відсутність менструації протягом року, швидку втомлюваність, гіпотонію з синкопальними станами зниження пам’яті, сухість шкіри. З анамнезу відомо, що 1,5 роки тому в II періоді пологів була значна кровотеча, проведено екстирпацію матки, гемотрансфузію, знаходилася на ШВЛ 3 доби. Який найбільш імовірний діагноз? A 29-year-old patient complains of the absence of menstruation for a year, rapid fatigue, hypotension with syncopal states, memory loss, dry skin. From the anamnesis, it is known that 1 5 years ago in the second stage of childbirth, there was significant bleeding, extirpation of the uterus, hemotransfusion was performed, she was on a ventilator for 3 days. What is the most likely diagnosis?

Адреногенітальний синдром Adrenogenital syndrome

Синдром Чіарі-Фромеля Chiari-Fromel syndrome

Синдром Фарбса-Олбрайта Farbs-Albright Syndrome

Синдром Шиєна Schien syndrome

Синдром Аронце дель Кастілліо Aronze del Castillo syndrome

126 / 200
При терміні гестації 32 тижні у вагітної почались передчасні пологи. Перейми через 10-15 хвилин, по 15-20 сек. Серцебиття плоду ясне, ритмічне - 145/хв. При піхвовому дослідженні шийка матки вкорочена, зовнішнє вічко розкрите на 1,5 см, плідний міхур цілий, передлежить голівка, виділення слизові, температура тіла 36,5oC. У крові: без відхилень. У мазку з піхви 4-5 лейкоцитів у п/з. Яка акушерська тактика? At a gestational age of 32 weeks, the pregnant woman went into premature labor. Contractions after 10-15 minutes, 15-20 seconds each. The fetal heartbeat is clear, rhythmic - 145/min During a vaginal examination, the cervix is shortened, the external eye is opened by 1.5 cm, the fetal bladder is intact, the head is present, mucous secretions, body temperature is 36.5oC. In the blood: no abnormalities. In a smear from the vagina, 4-5 leukocytes in n/ z. What obstetric tactics?

Спазмолітики, анальгетики Spasmolytics, analgesics

Токоліз. Профілактика дистрес-синдрому плоду Tocolysis. Prevention of fetal distress syndrome

Дати внутрішньовенний наркоз для зняття пологової діяльності Give intravenous anesthesia to relieve labor

Операція кесарського розтину Caesarean section operation

Пологи вести через природні пологові шляхи Give birth through the natural birth canal

127 / 200
Громадянин зловживає спиртними напоями, пропиває майно і заробітну плату, чим ставить себе, свою дружину і двох неповнолітніх дітей у важке матеріальне становище. Був на обліку в наркологічному диспансері за місцем проживання. Дружина звернулася до свого сімейного лікаря за порадою, з якою заявою до суду вона має право звернутися? The citizen abuses alcoholic beverages, drinks property and wages, which puts himself, his wife and two minor children in a difficult financial situation. He was registered in a drug dispensary for place of residence. The wife turned to her family doctor for advice on which application she has the right to apply to the court?

Про обмеження цивільної дієздатності чоловіка On the limitation of civil legal capacity of a man

Про визнання чоловіка неправоздатним On declaring a husband incompetent

Про визнання чоловіка недієздатним On declaring the husband incapable

Про визнання чоловіка частково дієздатним On recognition of the husband as partially capable

Про визнання чоловіка непрацездатним On declaring the husband unfit for work

128 / 200
Аналіз проб ґрунту, відібраних з шару 0-20 см, на земельній ділянці, відведеній під житлову забудову, показав, що кількість кишкових паличок (клітин в 1 г ґрунту) становить 15, ентерококів - 9, титр perfringens - 0,1, санітарне число - 0,99, нежиттєздатних форм яєць гельмінтів (екземплярів в 1 кг ґрунту) -5. Який з перелічених показників не відповідає гігієнічним нормативам і вказує на необхідність оздоровлення ґрунту на цій земельній ділянці? Analysis of soil samples taken from the 0-20 cm layer on a plot of land earmarked for residential development showed that the number of E. coli (cells in 1 g of soil ) is 15, enterococci - 9, perfringens titer - 0.1, sanitary number - 0.99, non-viable forms of helminth eggs (specimens in 1 kg of soil) - 5. Which of the listed indicators does not meet hygienic standards and indicates the need to improve the soil on this plot of land?

Титр perfringens Titer perfringens

Наявність нежиттєздатних форм яєць гельмінтів Presence of non-viable forms of helminth eggs

- -

Кількість кишкових паличок Number of E. coli

Кількість ентерококів Number of enterococci

129 / 200
Родина мешкає в районному центрі, що належить до зони радіаційного забруднення. Дитина шести років хворіла на ГРВІ протягом 19-ти днів. Лікувалася амбулаторно. За дитиною доглядала мама, працівниця кафе. Визначте порядок проведення експертизи непрацездатності: The family lives in the district center, which belongs to the zone of radiation contamination. A six-year-old child was ill with SARS for 19 days. She was treated on an outpatient basis. The child was looked after by her mother, cafe employee. Determine the procedure for conducting an examination of incapacity for work:

Листок непрацездатності видається на весь термін захворювання дитини The certificate of incapacity for work is issued for the entire duration of the child's illness

Листок непрацездатності видається на 7 днів, після чого довідка по догляду A certificate of incapacity for work is issued for 7 days, after which a certificate of care

Листок непрацездатності видається на 14 днів, після чого жодного документу не видається A certificate of incapacity for work is issued for 14 days, after which no document is issued

Листок непрацездатності не видається, тільки довідка по догляду на весь термін A certificate of incapacity for work is not issued, only a care certificate for the entire term

Листок непрацездатності видається загалом на 14 днів, після чого - довідка по догляду A certificate of incapacity for work is issued for a total of 14 days, after which a care certificate

130 / 200
Дитині 12 років. Скарги на тупий ниючий біль у епігастрії та правій підреберній ділянці, який посилюється після прийому жирної або смаженої їжі, головний біль, загальну слабкість, нудоту, підвищення температури тіла до субфебрильних цифр. При пальпації живота відмічається резистентність м’язів у правій підреберній ділянці, позитивні симптоми Кера, Ортнера, Мерфі. Який найбільш імовірний діагноз? The child is 12 years old. Complaints of a dull aching pain in the epigastrium and right subcostal region, which worsens after eating fatty or fried food, headache, general weakness, nausea, an increase in body temperature to subfebrile numbers. When palpating the abdomen, muscle resistance is noted in the right subcostal region, positive symptoms of Kerr, Ortner, Murphy. What is the most likely diagnosis?

Хронічний холецистит Chronic cholecystitis

Вірусний гепатит Viral hepatitis

Гострий апендицит Acute appendicitis

Гострий панкреатит Acute pancreatitis

Гострий гастрит Acute gastritis

131 / 200
У дівчинки 10-ти місяців, яка страждає на тривалий, з частими рецидивами, кашель, клінічно була запідозрена легенева форма муковісцидозу. Який діагностичний метод доцільно використати для його підтвердження? In a 10-month-old girl who suffers from a long-term, frequently recurring cough, the pulmonary form of cystic fibrosis was clinically suspected. What diagnostic method should be used to confirm it ?

Рентгенографія органів грудної клітки X-ray of chest organs

Копрограма Coprogram

Уреазний тест Urea test

Протеїнограма Proteinogram

Хлориди поту Sweat Chlorides

132 / 200
Хворого 47-ми років почав турбувати стискаючий біль за грудниною, що виникає під час ходи на 700-800 м. Один раз на тиждень випиває 2 л пива. Артеріальна гіпертензія впродовж останніх 7-ми років. Об’єктивно: Ps- 74/хв. АТ-120/80 мм рт.ст. При проведенні ВЕМ на навантаженні 75 Вт зареєстровано депресію сегмента ST на 2 мм нижче ізолінії у V4 — V6. Який найбільш імовірний діагноз? A 47-year-old patient began to be bothered by squeezing pain behind the sternum, which occurs during a walk of 700-800 m. He drinks 2 liters of beer once a week. Arterial hypertension for the past 7 years. Objectively: Ps-74/min. BP-120/80 mm Hg. When carrying out VEM at a load of 75 W, depression of the ST segment was registered 2 mm below the isoline in V4 — V6. What the most likely diagnosis?

Стенокардiя напруги, IV функціональний клас Tension angina, IV functional class

Стенокардія напруги, II функціональний клас Tension angina, II functional class

Вегето-судинна дистонія за гіпертонічним типом Vegeto-vascular dystonia of the hypertensive type

Алкогольна кардiомiопатiя Alcoholic cardiomyopathy

Стенокардiя напруги, III функціональний клас Tension angina, III functional class

133 / 200
-ти років хворіє з раннього дитинства. У крові: Нb- 110 г/л, ер.- 3,9-1012 133. /л, КП0,8, лейк.- 6,0-109 /л; ШЗЕ- 30 мм/год. Коагулограма: протромбіновий індекс 95%, ретракція кров’яного згортка 50%, час згортання крові - через 40 хвилин не відбулось, тривалість кровотечі - 3 хвилини. Які механізми лежать в основі патогенезу даного захворювання? -th years old sick since early childhood. Blood: Hb- 110 g/l, er.- 3.9-1012 133. /l, CP0, 8, leuk.- 6.0-109 /l; SZE- 30 mm/h. Coagulogram: prothrombin index 95%, retraction of blood clot 50%, blood clotting time - after 40 minutes did not occur, duration of bleeding - 3 minutes What mechanisms underlie the pathogenesis of this disease?

Недостатність у крові антигемофіль-ного глобуліну А Deficiency of antihemophilic globulin A in the blood

Дефіцит вітаміну С Vitamin C deficiency

!мунне пригнічення кісткового мозку !immune bone marrow suppression

Наявність специфічних антитіл до ендотеліальних стінок судин Presence of specific antibodies to the endothelial walls of vessels

Екзогенний дефіцит заліза Exogenous iron deficiency

134 / 200
Жінка 60-ти років протягом останнього року стала відзначати слабкість, запаморочення, швидку втомлюваність. Останнім часом - задишка, парестезії. Об’єктивно: шкіра та слизові оболонки бліді з іктеричним відтінком. Сосочки язика згладжені. Печінка, селезінка у краю реберної дуги. У крові: Hb- 70 г/л, ер.- 1,7 • 1012/л, КП- 1,2, макроцити. Призначення якого препарату є патогенетично обґрунтованим? During the last year, a 60-year-old woman began to note weakness, dizziness, rapid fatigue. Recently, shortness of breath, paresthesias. Objectively: the skin and mucous membranes are pale with an icteric tinge. The papillae of the tongue are smoothed. Liver, spleen at the edge of the costal arch. In the blood: Hb- 70 g/l, er.- 1.7 • 1012/l, KP- 1.2, macrocytes. What drug is prescribed pathogenetically justified?

Вітамін B12 Vitamin B12

Вітамін В6 Vitamin B6

Препарати заліза Iron preparations

Вітамін В1 Vitamin B1

Аскорбінова кислота Ascorbic acid

135 / 200
Чоловік 27-ми років скаржиться на задишку, свербіж і відчуття набряку обличчя, що виникли 25 хвилин тому, Їв раків та пив пиво. Стан прогресивно погіршується. Об’єктивно: збуджений. Обличчя одутле, щоки та губи збільшені у розмірі. ЧД- 28/хв., чутні свистячі хрипи на вдиху та видиху, при аускультації легень провідні шуми, максимум над грудниною. Тони серця гучні, ЧСС=Р8=108/хв., АТ- 150/90 мм рт.ст. Найбільш імовірною причиною задишки є: A 27-year-old man complains of shortness of breath, itching, and a feeling of facial swelling that occurred 25 minutes ago. He ate crayfish and drank beer. The condition is getting progressively worse. About' objectively: excited. The face is swollen, the cheeks and lips are enlarged. BH- 28/min., whistling rales are audible on inhalation and exhalation, when auscultating the lungs, conducting noises, maximum above the sternum. Heart sounds are loud, heart rate = P8 = 108/min ., blood pressure - 150/90 mm Hg. The most likely cause of shortness of breath is:

Набряк гортані Swelling of the larynx

Алкогольне сп’яніння Alcohol intoxication

Спазм бронхів Bronchi spasm

Нейроциркуляторна дистонія Neurocirculatory dystonia

Гіпертонічний криз Hypertensive crisis

136 / 200
Аналіз організації медичної допомоги у обласному центрі показав, що кожен рік біля 12% пацієнтів отримують стаціонарну допомогу при захворюваннях, що не потребують цілодобового нагляду та інтенсивного догляду. Які організаційні перетворення найдоцільніше провести для вирішення даної проблеми? Analysis of the organization of medical care in the regional center showed that every year about 12% of patients receive inpatient care for diseases that do not require round-the-clock supervision and intensive care. What organizational transformation is the most expedient to carry out to solve this problem?

Зміна статуту амбулаторно-поліклінічних закладів Change in the charter of outpatient polyclinic institutions

Структурна перебудова спеціалізованої допомоги Structural restructuring of specialized assistance

Розвиток стаціонарозамінюючих видів допомоги Development of inpatient replacement types of assistance

Розвиток первинної медико-санітарної допомоги Development of primary healthcare

Зміцнення матеріально-технічної бази стаціонарів Strengthening the material and technical base of inpatients

137 / 200
Доношений новонароджений з масою тіла 4500 г народився в асфіксії з оцінкою за шкалою Апгар 4-6 балів. В пологах утруднене виведення плечового поясу. У неврологічному статусі загальномозкові розлади, виявлений тотальний верхній млявий парез - рука атонічна, пронована, не викликаються рефлекси - хапальний, Бабкіна, Моро. Укажіть сегменти ураження спинного мозку: A full-term newborn with a body weight of 4500 g was born asphyxiated with an Apgar score of 4-6 points. During delivery, the removal of the shoulder girdle is difficult. In the neurological status, general brain disorders, revealed total upper flaccid paresis - the hand is atonic, pronated, reflexes are not evoked - grasping, Babkina, Moro. Specify the segments of the spinal cord lesion:

СV - Th1 СV - Th1

ThI - ThV ThI - ThV

ThVI - ThVII ThVI - ThVII

СIII - СIV CIII - CIV

СI - СII СИ - СII

138 / 200
Жінка 57-ми років скаржиться на відчуття стиснення у стравоході, серцебиття, утруднення дихання при прийомі твердої їжі, іноді з’являється блювання повним ротом, вночі - симптом 'мокрої подушки'. Хворіє близько 6-ти місяців. Об’єктивно: температура - 39oC, зріст -168 см, вага - 72 кг, Ps- 76/хв., АТ-120/80 мм рт.ст. Рентгенологічно: стравохід значно розширений, у кардіальній частині - звужений. Яка патологія найбільш імовірно викликала дисфагію у хворої? A 57-year-old woman complains of a feeling of compression in the esophagus, palpitations, difficulty breathing when eating solid food, sometimes vomiting with a full mouth appears, at night - a symptom ' wet pillow'. He has been ill for about 6 months. Objectively: temperature - 39oC, height -168 cm, weight - 72 kg, Ps - 76/min., BP - 120/80 mm Hg. Radiologically: the esophagus is significantly expanded, in the cardiac part - narrowed. What pathology most likely caused dysphagia in the patient?

Рак стравоходу Esophageal cancer

Первинний езофагоспазм Primary esophageal spasm

Ахалазія кардії Achalasia cardia

Грижа стравохідного отвору дiафра-гми Hernia of the esophageal opening of the diaphragm

Рефлюкс-езофагіт Reflux-esophagitis

139 / 200
Хворий 62-х років скаржиться на втрату 10 кг ваги за 2 місяці, появу сєчі кольору 'м’ясних помиїв', біль у поперековій ділянці, підвищення температури до 39oC. При пальпації лівої нирки скаржиться на біль. У крові: ер.-2,8 • 1012/л, Hb- 90 г/л, лейк.- 8,8 • 109 /л, ШОЕ- 42 мм/год. У сечі: питома вага -1018, білок - 0,66 г/л, еритроцити на все поле зору. Який попередній діагноз? A 62-year-old patient complains of a 10 kg weight loss in 2 months, the appearance of urine the color of 'meat slops', pain in the lumbar region, an increase in temperature to 39oC. When palpating the left kidney, he complains of pain. In the blood: ER - 2.8 • 1012/l, Hb - 90 g/l, leuk. - 8.8 • 109 /l, ESR - 42 mm/h. urine: specific gravity -1018, protein - 0.66 g/l, erythrocytes in the entire field of view. What is the previous diagnosis?

Гострий гломерулонефрит Acute glomerulonephritis

Карбункул нирки Kidney carbuncle

Туберкульоз нирки Kidney tuberculosis

Пухлина нирки Kidney tumor

Сечокам’яна хвороба Urolithiasis

140 / 200
У пацієнта 54-х років м’яка первинна артеріальна гіпертензія, ІХС: стенокардія напруги II ФК, СН II ст. Супутній діагноз: гастроезофагеальна рефлюксна хвороба, ерозивний езофагіт II стадія. Постійний прийом якого з препаратів може викликати у хворого посилення проявів гастроентерологічної патології? A 54-year-old patient has mild primary arterial hypertension, coronary artery disease: angina pectoris of the II FC, HF of the II stage. Associated diagnosis: gastroesophageal reflux disease, erosive esophagitis Stage II. The constant intake of which of the drugs can cause the patient to increase the manifestations of gastroenterological pathology?

Еналаприлу малеат Enalapril Maleate

Гідрохлортіазид Hydrochlorothiazide

Метопролол Metoprolol

Ізосорбіду динітрат Isosorbide dinitrate

Омепразол Omeprazole

141 / 200
Хворому 78-ми років з аденомою передміхурової залози виконано грижосічення з приводу прямої пахвинної грижі. Після операції відсутнє сечовипускання. Над лоном визначається збільшений сечовий міхур. Що необхідно виконати? A 78-year-old patient with an adenoma of the prostate gland underwent a hernia resection for a direct inguinal hernia. After the operation, there is no urination. An enlarged bladder is detected above the pubis. What needs to be done?'

Покласти на ділянку сечового міхура холод Put cold on the bladder area

Призначити внутрішньом’язово прозерин Prescribe intramuscular proserin

Призначити УВЧ на післяопераційну рану Prescribe UHF for postoperative wound

Призначити підшкірно спазмолітики Prescribe antispasmodics subcutaneously

Катетеризація сечового міхура Bladder catheterization

142 / 200
На першу добу після операції з приводу дифузного токсичного зобу у хворої виникли скарги на утруднення дихання, холодний піт, слабкість. Об’єктивно: шкіра бліда, температура тіла 38,5oC, ЧДР- 25/хв., Ps- 110/хв., АТ- 90/60 мм рт.ст. Яке ускладнення раннього післяопераційного періоду розвинулося у хворої? On the first day after surgery for diffuse toxic goiter, the patient complained of difficulty breathing, cold sweat, weakness. Objectively: pale skin, body temperature 38 ,5oC, ChDR- 25/min., Ps- 110/min., BP- 90/60 mm Hg. What complication of the early postoperative period developed in the patient?

Гіпотиреоїдний криз Hypothyroid crisis

Післяопераційна тетанія Postoperative tetany

Тиреотоксичний криз Thyrotoxic crisis

Гострий тиреоїдит Acute thyroiditis

Стиснення трахеї гематомою Compression of trachea by hematoma

143 / 200
Першовагітну госпіталізовано зі скаргами на головні болі. Жіночу консультацію не відвідувала. Встановлено термін вагітності 35-36 тижнів. АТ-180/120 мм рт.ст. праворуч, 140/90 мм рт.ст. ліворуч, набряки нижніх і верхніх кінцівок. У сечі: білок 3,97 г/л, гіалінові та зернисті циліндри. Який найбільш імовірний діагноз? The first-time pregnant woman was hospitalized with complaints of headaches. She did not attend the women's consultation. The gestational age was determined to be 35-36 weeks. BP-180/120 mm Hg on the right, 140/90 mm Hg on the left, edema of the lower and upper extremities. In the urine: protein 3.97 g/l, hyaline and granular cylinders. What is the most likely diagnosis?

Прееклампсія середнього ступеня Moderate preeclampsia

Прееклампсія легкого ступеня Mild preeclampsia

HELLP-синдром HELLP syndrome

Поєднаний НПГ-гестоз Combined NPG-gestosis

Прееклампсія тяжкого ступеня Severe preeclampsia

144 / 200
Хвора 45-ти років висловлює скарги на головний біль, серцебиття, стискаючий біль за грудниною. Протягом семи років хворіє на гіпертонічну хворобу. Під час лікування з’явились набряки нижніх кінцівок. Лікар припустив побічну дію ліків. Який з перерахованих препаратів міг викликати таку реакцію? A 45-year-old patient complains of headache, palpitations, squeezing pain behind the sternum. She has been suffering from hypertension for seven years. During the treatment, edema appeared lower limbs. The doctor assumed a side effect of the drugs. Which of the listed drugs could have caused such a reaction?

Небіволол Nebivolol

Лізиноприл Lisinopril

фбесартан fbesartan

Метопролол Metoprolol

Фелодипін Felodipine

145 / 200
Робітниця заводу з обробки шкіри, стаж роботи у контакті із урсолом 12 років, скаржиться на виражений свербіж шкіри. Об’єктивно: на кистях і пальцях рук, передпліччях, обличчі і шиї наявні симетричні папульозні поліморфні висипання. Стан погіршується після роботи, у вихідні дні та під час відпустки почуває себе краще. Що із переліченого слід застосувати у даному випадку? A worker at a leather processing plant, 12 years of work experience in contact with Ursol, complains of pronounced itching of the skin. Objectively: on the hands and fingers, forearms, there are symmetrical papular polymorphic rashes on the face and neck. The condition worsens after work, on weekends and during vacation, he feels better. Which of the following should be used in this case?

Антигістамінні препарати Antihistamines

Радіоактивні ізотопи Radioactive isotopes

Рентгенотерапія X-ray therapy

Дезинфікуючі розчини Disinfectant solutions

Препарати сірки Sulphur drugs

146 / 200
Хвора 45-ти років доставлена машиною швидкої допомоги зі скаргами на різку загальну слабкість, нудоту, блювання, біль у животі. Останнім часом відзначає зниження апетиту, схуднення. Об’єктивно: гіперпігментація шкіри, АТ- 70/45 мм рт.ст., брадикардія. В додаткових дослідженнях знижений вміст альдостерону, кортизолу в крові, знижена екскреція 17-КС та 17-ОКС з сечею, гіпонатріємія, гіпохлоремія, гіпокаліємія. Які лікувальні заходи потрібно вжити? A 45-year-old patient was brought by ambulance with complaints of severe general weakness, nausea, vomiting, abdominal pain. Recently, she has noted a decrease in appetite, weight loss. About objectively: hyperpigmentation of the skin, blood pressure - 70/45 mm Hg, bradycardia. In additional studies, the content of aldosterone and cortisol in the blood is reduced, the excretion of 17-KS and 17-OKS in the urine is reduced, hyponatremia, hypochloremia, hypokalemia. measures should be taken?

Призначення глюкокортикоїдів, мі-нералокортикоїдів, дієти з підвищеним вмістом кухарської солі Prescription of glucocorticoids, mineralocorticoids, diet with a high content of table salt

Призначення інсуліну Prescription of insulin

Призначення дієтотерапії з підвищеним вмістом кухарської солі Prescription of dietary therapy with an increased content of table salt

Призначення альдостерону Aldosterone Prescription

Призначення преднізолону Prednisone Prescription

147 / 200
Хворий 35-ти років скаржиться на біль у верхній третині плеча, який посилюється вночі. Об’єктивно: помірний набряк у верхній третині плеча, шкіра над ним підвищеної температури, болісність під час пальпації, обмеження рухів у плечовому суглобі. На рентгенограмах - ділянка деструкції плечової кістки у метадіафізарному відділі з явищами гомілкового періоститу (спікули) і відшаруванням окістя у вигляді 'дашка'. Встановіть попередній діагноз: A 35-year-old patient complains of pain in the upper third of the shoulder, which worsens at night. Objectively: moderate swelling in the upper third of the shoulder, the skin over it has a high temperature , pain during palpation, limitation of movements in the shoulder joint. X-rays show an area of destruction of the humerus in the metadiaphyseal region with signs of tibial periostitis (spicules) and peeling of the periosteum in the form of a 'dashka'. Establish a preliminary diagnosis:

Остеогенна саркома Osteogenic sarcoma

Гемангіома Hemangioma

Хондробластома Chondroblastoma

Хондрома Chondroma

Остеома Osteoma

148 / 200
На 8-му добу життя у дитини, яка народилась у строк, з масою 3500 г, піднялась температура тіла до 37,5oC, стала мляво брати груди. Дитина була прикладена до грудей матері на 3-тю добу в зв’язку з післяпологовим ендометритом у матері. Об’єктивно: дитина активна. На шкірі грудей, живота, стегон виявлені поверхневі в’ялі пухирі числом 10, діаметром 5-10 мм, з мутним вмістом. Деякі з них зруйнувались, при цьому з’явилась яскраво-рожева поверхня. З боку внутрішніх органів змін немає. Вкажіть найбільш імовірний діагноз: On the 8th day of life, a child who was born at term, weighing 3500 g, had a body temperature of 37.5oC, began to breastfeed sluggishly. The child was applied to the mother's breast on the 3rd day in connection with postpartum endometritis in the mother. Objectively: the child is active. On the skin of the chest, abdomen, thighs, surface flaccid blisters number 10, 5-10 mm in diameter, with cloudy contents. Some of them have collapsed, while a bright pink surface has appeared. There are no changes in the internal organs. Specify the most likely diagnosis:

Ексфоліативний дерматит Ріттера Ritter's exfoliative dermatitis

Везикулопустульоз Vesiculopustulosis

Інфікована попрілість Infected health

Пухирчатка новонароджених Neonatal pemphigus

Синдром Лайєла Lyell syndrome

149 / 200
При профілактичному ультразвуковому обстеженні органів черевної порожнини в середній школі у учениці 5-го класу (11 років) ліва нирка візуалізується на 3 см нижче за норму, нормальних розмірів, форми та структури, контрлатеральна нирка в типовому місті не виявлена. Попередній діагноз: уроджена аномалія розвитку нирок, дистопія лівої нирки, відсутність правої нирки або її тазова дистопія. Який метод променевої діагностики доцільно застосувати для встановлення остаточного діагнозу та визначення функціональної здатності обох нирок? During a preventive ultrasound examination of the abdominal cavity in a secondary school student of the 5th grade (11 years old), the left kidney is visualized 3 cm below the norm, of normal size, shape and structure, the contralateral kidney in a typical city is not detected. Preliminary diagnosis: congenital anomaly of kidney development, dystopia of the left kidney, absence of the right kidney or its pelvic dystopia. What radiological diagnostic method should be used to establish the final diagnosis and determine the functional capacity of both kidneys?

Радіоіммуний аналіз Radioimmune analysis

Радіонуклідна ренографія Radionuclide renography

Динамічна реносцинтіграфія Dynamic renoscintigraphy

Екскреторна урографія Excretory urography

Термографія Thermography

150 / 200
У хворої 54-х років на оглядовій рентгенограмі органів грудної порожнини зліва паратрахеально виявлені множинні утворення розмірами від 2 до 4 см в діаметрі. Скарги на покашлювання та задишку при незначних фізичних навантаженнях. В анамнезі рак молочної залози після радикальної терапії 5 років тому. Діагноз: пролонгація хвороби, метастатичне ураження легень. Яку тактику терапії слід обрати? In a 54-year-old patient, multiple formations of 2 to 4 cm in diameter were found on the radiograph of the chest cavity on the left paratracheal. Complaints of coughing and shortness of breath with minor physical exertion. There is a history of breast cancer after radical therapy 5 years ago. Diagnosis: prolongation of the disease, metastatic lung damage. What treatment tactics should be chosen?

Операційне втручання Operational intervention

Протизапальна антибактеріальна терапія Anti-inflammatory antibacterial therapy

Радикальна хіміотерапія Radical chemotherapy

Радикальна променева терапія Radical radiotherapy

Паліативна хіміо- та променева терапія Palliative chemotherapy and radiotherapy

151 / 200
Лікар швидкої допомоги прибув на виклик до особи, яку родичі витягли із зашморгу. Об’єктивно: відсутність пульсу на сонних артеріях, відсутність свідомості, самостійного дихання, корнеальних рефлексів та наявність трупних плям на спині і задній поверхні кінцівок. За якими ознаками можна констатувати настання смерті? The emergency doctor arrived on call to a person who was pulled out of the bed by relatives. Objectively: lack of pulse on the carotid arteries, lack of consciousness, independent breathing, corneal reflexes and the presence of cadaveric spots on the back and back surface of the limbs. What signs can be used to determine the onset of death?

Відсутність самостійного дихання Lack of spontaneous breathing

Відсутність пульсу No pulse

Відсутність корнеальних рефлексів Absence of corneal reflexes

Відсутність свідомості Lack of consciousness

Наявність трупних плям Presence of corpse stains

152 / 200
Хворий 48-ми років звернувся до лікаря зі скаргами на біль у попереку. Хворіє 3 дня після переохолодження. Після огляду лікар поставив діагноз: гострий попереково-крижовий радикуліт. Які ліки слід призначити хворому? A 48-year-old patient turned to the doctor with complaints of lower back pain. He has been ill for 3 days after hypothermia. After the examination, the doctor diagnosed acute lumbosacral sciatica. What medicines should be prescribed to the patient?

Антибактеріальні Antibacterial

Кортикостероїди Corticosteroids

Десенсибілізуючі Desensitizing

Нестероїдні протизапальні засоби Nonsteroidal anti-inflammatory drugs

Вітаміни Vitamins

153 / 200
Хворий 67-ми років перебуває на стаціонарному лікуванні у протитуберкульозному диспансері впродовж двох місяців з приводу рецидиву туберкульозу (24.02.2005) S6 лівої легені (інфільтративний). Хворому призначено наступне лікування: ізоніазид + рифампіцин + стрептоміцин + піразинамід + етамбутол. У хворого з’явились скарги на диплопію, обмеження поля зору. Який з вищезазначених препаратів викликав таку побічну дію? A 67-year-old patient has been undergoing inpatient treatment in an anti-tuberculosis dispensary for two months due to a recurrence of tuberculosis (February 24, 2005) S6 of the left lung (infiltrative). The patient was prescribed the following treatment: isoniazid + rifampicin + streptomycin + pyrazinamide + ethambutol. The patient complained of diplopia, limitation of the field of vision. Which of the above drugs caused such a side effect?

Стрептоміцин Streptomycin

Ізоніазид Isoniazid

Піразинамід Pyrazinamide

Рифампіцин Rifampicin

Етамбутол Etambutol

154 / 200
Повторновагітна 24-х років з Rh-негативним типом крові знаходиться під наглядом перинатолога. В анамнезі: у попередніх пологах проведене ручне відділення плаценти з приводу кровотечі у III періоді. В 36 тижнів вагітності підвищився титр антитіл з 1:16 до 1:64. При УЗД - потовщення плаценти і сповільнення рухів плода. З якою частотою в подальшому необхідно проводити дослідження крові на Rh-антитіла? A 24-year-old re-pregnant woman with Rh-negative blood type is under the supervision of a perinatologist. In the anamnesis: in previous deliveries, the placenta was manually separated due to bleeding in the III period. At 36 weeks of pregnancy, the titer of antibodies increased from 1:16 to 1:64. At ultrasound, there is thickening of the placenta and slowing of fetal movements. With what frequency in the future is it necessary to conduct a blood test for Rh antibodies?

Щоденно до розродження Every day before giving birth

1 раз на 2 тижні 1 time in 2 weeks

1 раз на 3 тижні 1 time in 3 weeks

1 раз на тиждень Once a week

Перед пологами Before giving birth

155 / 200
Хвора 49-ти років госпіталізована зі скаргами на слабкість, жовтяницю, свербіж шкіри. Захворіла 2,5 місяці тому. При госпіталізації - значна жовтяниця. Печінка не пальпується. Жовчний міхур збільшений, безболісний. Білірубін крові -190 мкмоль/л, за рахунок прямого. Кал ахолічний. Яка найбільш імовірна причина жовтяниці? A 49-year-old patient was hospitalized with complaints of weakness, jaundice, itchy skin. She became ill 2.5 months ago. During hospitalization, significant jaundice. The liver is not palpable. The gall bladder is enlarged, painless. Blood bilirubin -190 μmol/l, due to direct. Stool is acholic. What is the most likely cause of jaundice?

Гемолітична жовтяниця Hemolytic Jaundice

Синдром Каролі Caroli Syndrome

Паренхіматозна жовтяниця Parenchymal jaundice

Хвороба Жильбера Gilbert's disease

Механічна жовтяниця Mechanical jaundice

156 / 200
В операційній обласної клінічної лікарні проведено вимірювання мікрокліматичних показників. Результати проведених досліджень: середня температура повітря складає 22oC, відносна вологість повітря - 48%, швидкість руху повітря - 0,1 м/с. Дайте гігієнічну оцінку мікроклімату операційної: In the operating room of the regional clinical hospital, microclimatic indicators were measured. The results of the conducted research: the average air temperature is 22oC, the relative humidity of the air is 48%, the speed of air movement is 0, 1 m/s Give a hygienic assessment of the microclimate of the operating room:

Мікроклімат дискомфортний The microclimate is uncomfortable

Мікроклімат комфортний The microclimate is comfortable

Мікроклімат дискомфортний з підвищеною вологістю The microclimate is uncomfortable with high humidity

Мікроклімат дискомфортний охолоджуючий The microclimate is uncomfortable cooling

Мікроклімат дискомфортний з підвищеною швидкістю руху повітря The microclimate is uncomfortable with increased air movement speed

157 / 200
-річної дитини протягом 6-ти місяців періодично з’являється лихоманка до 38,5o 157. C, кашель, задишка, одноразово кровохаркання. БЦЖ-рубець відсутній. Дідусь хворіє на туберкульоз. В легенях - розсіяні сухі та різнокаліберні вологі хрипи. Рентгенологічно: однотипна рівномірна дрібновогнищева інфільтрація легень, у верхніх частках тонкостінна каверна із слабковираженою перифокальною інфільтрацією. Яке захворювання найбільш імовірне? -year-old child periodically has a fever up to 38.5o 157. C, cough, shortness of breath, one-time hemoptysis for 6 months. There is no BCG scar. Grandfather is sick with tuberculosis. There are scattered dry and wet rales of various calibers in the lungs. X-ray: the same type of uniform small-focal infiltration of the lungs, in the upper lobes a thin-walled cavity with weak perifocal infiltration. What disease is most likely?

Бронхіальна астма Bronchial asthma

Вогнищева пневмонія Focal pneumonia

Хронічний бронхіт Chronic bronchitis

Синдром Хамана-Річа Hamann-Rich Syndrome

Туберкульоз легень Pulmonary tuberculosis

158 / 200
Гірничий робітник очисного вибою 37-ми років після довготривалого вимушеного зігнутого положення тулуба у шахті відчув інтенсивний, стріляючого характеру біль у поперековому відділі хребта, який поширювався вниз у ліву ногу до підколінної ямки. Рухи хребта у поперековій ділянці різко обмежені. Позитивний симптом Ласега зліва. Пальпаторна болючість паравертебральних точок L5 - S1 . Сухожилкові рефлекси на нижніх кінцівках - знижений лівий ахіловий рефлекс. Гіпотонія м’язів лівого стегна і гомілки. Встановіть попередній клінічний діагноз: A 37-year-old mining worker of the cleaning pit, after a long-term forced bent position of the trunk in the mine, felt an intense, shooting pain in the lumbar spine, which spread down to the left leg to the popliteal fossa. Movements of the spine in the lumbar region are sharply limited. Positive Laseg symptom on the left. Palpatory tenderness of the paravertebral points L5 - S1. Tendon reflexes on the lower limbs - reduced left Achilles reflex. Hypotonia of the muscles of the left thigh and lower leg. Establish a preliminary clinical diagnosis:

Транзиторна ішемічна атака Transient ischemic attack

Перелом поперекового хребця Lumbar fracture

Попереково-крижова радикулопатія Lumbosacral radiculopathy

Ниркова коліка Renal colic

Спінальний інсульт Spinal stroke

159 / 200
Хворий 18-ти років скаржиться на припухлість правого гомілковоступневого суглоба, обмеження рухливості, болі, що посилюються під час ходи. В анамнезі 'віраж' туберкулінових проб у 7-річному віці. Лікування не приймав через відмову батьків. Рентгенологічно: дистальне епіметафізне вогнище з деструкцією епіфізу великогомілкової кістки, суглобова щілина різко звужена. Реакція Манту з 2 ТО- 18 мм. Який найбільш імовірний діагноз? An 18-year-old patient complains of swelling of the right ankle joint, limitation of mobility, pain that worsens when walking. In the anamnesis, tuberculin tests have been 'curved' in 7- at the age of one year. He did not receive treatment due to the refusal of his parents. X-ray: distal epimetaphyseal focus with destruction of the tibial epiphysis, the joint space is sharply narrowed. Mantoux reaction with 2 TO - 18 mm. What is the most likely diagnosis?

Сифіліс правого гомілковоступнево-го суглоба Syphilis of the right ankle joint

Травматичний артрит Traumatic arthritis

Туберкуліновий остит правого гоміл-ковоступневого суглоба Tuberculin osteitis of the right ankle-foot joint

Саркома правого гомілковоступнево-го суглоба Sarcoma of the right ankle-foot joint

Епіфізарний остеомієліт Epiphyseal osteomyelitis

160 / 200
Дитина 5-ти років півроку тому була оперована з приводу УВС. Скарги на лихоманку впродовж 3-х тижнів, біль у ділянці серця, м’язах та кістках. При огляді - шкіра кольору 'кави з молоком', аускультативно - систолічний шум в ділянці серця з шумом в III-IV міжребер’ї. На кінчиках пальців встановлено плями Джейнуеля. Який попередній діагноз? A 5-year-old child six months ago was operated on for UBS. Complaints of fever for 3 weeks, pain in the area of the heart, muscles and bones. On examination - the skin is the color of 'coffee with milk', auscultation - a systolic murmur in the heart area with a murmur in the III-IV intercostal space. On the tips of the fingers, Jainuel's spots have been established. What is the preliminary diagnosis?

Інфекційний ендокардит Infective endocarditis

Черевний тиф Typhoid

Сепсис Sepsis

Гостра ревматична лихоманка Acute rheumatic fever

Неревматичний кардит Non-rheumatic carditis

161 / 200
Третя доба після 1-х термінових нормальних пологів. Дитина на сумісному перебуванні, природньому вигодовуванні. Об’єктивно: загальний стан задовільний. Температура 36,4oC, Ps-80/хв., АТ- 120/80 мм рт.ст. Молочні залози м’які, безболісні. Лактація помірна, відтік молока не утруднений. Матка щільна, дно матки на 3 п/п нижче пупка. Лохії кров’янистосерозні, у помірній кількості. Оцініть динаміку зворотного розвитку матки: The third day after the 1st term normal delivery. The child is in a joint stay, natural feeding. Objectively: the general condition is satisfactory. Temperature 36.4oC, Ps- 80/min., blood pressure - 120/80 mm Hg. Mammary glands are soft, painless. Lactation is moderate, milk outflow is not difficult. Uterus is dense, the bottom of the uterus is 3 p/p below the navel. Lochia is blood-serous, in a moderate amount. Evaluate the dynamics of the reverse development of the uterus:

Субінволюція Subinvolution

Іематометра Hematometer

Фізіологічна інволюція Physiological involution

Патологічна інволюція Pathological involution

Лохіометра Lochiometer

162 / 200
У електрозварювальника зі стажем роботи 15 років під час медичного огляду виявлено сухі хрипи в нижніх відділах легень. На рентгенограмі спостерігаються дифузні вузлики розміром 3-4 мм в середніх і нижніх відділах легень. Яке захворювання можна запідозрити? During a medical examination, an electric welder with 15 years of experience was found to have dry wheezing in the lower parts of the lungs. On the radiograph, diffuse nodules of 3-4 mm in size are observed in the middle and lower lung departments. What disease can be suspected?

Силікоз Silicosis

Бронхіт Bronchitis

Силікатоз Silicatosis

Металоконіоз Metaloconiosis

Карбоконіоз Carboconiosis

163 / 200
Сімейний лікар на виклику засвідчив смерть. На підставі якого медичного документу реєструється цей випадок смерті та в який максимальний термін має відбутись реєстрація смерті в РАГ-Сі? The on-call family doctor certified the death. On the basis of which medical document is this case of death registered and in what maximum period should the death be registered in the RAG-C?

Лікарське свідоцтво про смерть. Термін -1 місяць Medical death certificate. Term -1 month

Медична довідка про смерть. Термін -3 доби Medical death certificate. Term -3 days

Лікарське свідоцтво про смерть. Термін - 7 діб Medical death certificate. Term - 7 days

Лікарське свідоцтво про смерть. Термін - 3 доби Medical death certificate. Term - 3 days

Медична довідка про смерть. Термін -3 місяці Medical death certificate. Term - 3 months

164 / 200
Хворий 19-ти років скаржиться на серцебиття та задуху при фізичному навантаженні. Об’єктивно: межі серця зміщені вправо. При аускультації вислуховуються систолічний шум у II-III міжребер’ї зліва від груднини, акцент II тону на легеневій артерії, іноді шум Грехем-Стіла. На ЕКГ фіксуються ознаки гіпертрофії правого шлуночка та блокада правої ніжки пучка Гіса. Яка найбільш імовірна патологія зумовила таку картину? A 19-year-old patient complains of palpitations and shortness of breath during physical exertion. Objectively: the borders of the heart are shifted to the right. During auscultation, a systolic murmur is heard in the II-III intercostal space to the left of the sternum, an accent of the II tone on the pulmonary artery, sometimes a Graham-Steele noise. The ECG shows signs of right ventricular hypertrophy and blockade of the right leg of the bundle of His. What is the most likely pathology that caused such a picture?

Передсердний септальний дефект Atrial septal defect

Тетрада Фалло Tetrad of Fallot

Дефект міжшлуночкової перегородки Ventricular septal defect

Стеноз легеневої артерії Pulmonary artery stenosis

Відкрита артеріальна протока Open ductus arteriosus

165 / 200
Пацієнтка 35-ти років звернулась до лікаря жіночої консультації з питанням: що можна зробити для настання вагітності. Із анамнезу: перенесла дві операції з приводу позаматкової вагітності, обидві маткові труби видалені при операціях. Які можливі методи лікування безпліддя у жінки слід застосувати? A 35-year-old patient turned to the gynecologist with the question: what can be done to get pregnant. From the anamnesis: she underwent two operations for ectopic pregnancy, both uterine tubes removed during operations. What possible methods of treatment for female infertility should be used?

Екстракорпоральне запліднення In vitro fertilization

Сурогатне материнство Surrogacy

Інсемінація спермою чоловіка Insemination with male sperm

Індукція овуляції Ovulation induction

Оперативна лапароскопія Operative laparoscopy

166 / 200
Хворий страждає на дисемінований туберкульоз легень 5 років. Останнім часом з’явилися біль у поперековій ділянці, часті сечовипускання. У сечі: кисла реакція, протєїнурія; при бактеріоскопії мазка сечі - кислотостійкі бактерії. Яке ураження нирок у цього хворого? The patient has been suffering from disseminated tuberculosis of the lungs for 5 years. Recently, pain in the lumbar region appeared, frequent urination. In urine: acidic reaction, proteinuria; during bacterioscopy, smear urine - acid-resistant bacteria. What kind of kidney damage does this patient have?

Амілоїдоз Amyloidosis

Гломерулонефрит Glomerulonephritis

Пієлонефрит Pyelonephritis

Гідронефроз Hydronephrosis

Туберкульоз нирок Kidney tuberculosis

167 / 200
Жінка 30-ти років звернулася до лікаря зі скаргами на свербіж шкіри, який посилюється увечері і турбує її вже тиждень. Патологічний процес поширюється з кистей на шкіру живота, сідниць, лобка. Об’єктивно: поліморфний висип, що складається з дрібних парних міхурців, цяткових папул, екскоріацій, розчухів, запальних папул червоно-коричневого кольору. Який найбільш імовірний діагноз? A 30-year-old woman consulted a doctor with complaints of skin itching, which worsens in the evening and has been bothering her for a week. The pathological process spreads from the hands to the skin of the abdomen, buttocks , pubis. Objectively: a polymorphic rash consisting of small paired vesicles, speckled papules, excoriations, scratches, inflammatory red-brown papules. What is the most likely diagnosis?

Нейродерміт Neurodermatitis

Педикульоз Pediculosis

Контактно-алергійний дерматит Contact-allergic dermatitis

Короста Scabies

Дисгідротична екзема Dyshidrotic eczema

168 / 200
У населеному пункті планується будівництво багатопрофільної лікарні на 500 ліжок. Де на лікарняній ділянці необхідно розмістити поліклініку? The construction of a multidisciplinary hospital with 500 beds is planned in the settlement. Where on the hospital site should the polyclinic be located?

Біля господарської зони Near the economic zone

У центрі ділянки біля лікувальних корпусів In the center of the site near the medical buildings

Біля центрального входу Near the central entrance

У садово-парковій зоні In the park zone

Розміщення поліклініки на території ділянки не дозволяється Placing a polyclinic on the site is not allowed

169 / 200
При вивченні санітарно-гігієнічних умов у 4-х ліжковій терапевтичній палаті встановлено: площа палати - 30 м2 , висота - 3,2 м, температура повітря +20oC, вологість - 55%, швидкість руху повітря - 0,1 м/с, світловий коефіцієнт (СК) - 1:5, коефіцієнт природної освітленості (КПО) - 0,6%, вміст діоксиду вуглецю у повітрі - 0,1%. Який з показників не відповідає гігієнічним вимогам? When studying the sanitary and hygienic conditions in the 4-bed therapeutic ward, the following was established: ward area - 30 m2, height - 3.2 m, air temperature +20oC, humidity - 55%, air movement speed - 0.1 m/s, light ratio (SC) - 1:5, natural light ratio (LDL) - 0.6%, carbon dioxide content in the air - 0.1%. What from indicators does not meet hygienic requirements?

Коефіцієнт природної освітленості (КПО) Coefficient of natural illumination (KPO)

Світловий коефіцієнт (СК) Light factor (LC)

Площа палати Room area

Швидкість руху повітря Air speed

Вміст діоксиду вуглецю у повітрі Content of carbon dioxide in the air

170 / 200
Вагітна 25-ти років в терміні 38- 39 тижнів, яка проживає в сільській місцевості, звернулась до акушерки сільського ФАПу із скаргами на переймоподібні болі в низу живота та поперековій ділянці. Яку допомогу має надати медпрацівник? A 25-year-old pregnant woman at 38-39 weeks, who lives in a rural area, turned to the midwife of the rural FAP with complaints of cramp-like pains in the lower abdomen and lumbar area. What help should a medical worker provide?

Ввести спазмолітичний засіб Enter antispasmodic

Заспокоїти жінку Calm the woman

Госпіталізувати жінку в обласну лікарню, пологове відділення Hospitalize the woman in the regional hospital, maternity ward

Госпіталізувати жінку в районну лікарню, пологове відділення Hospitalize the woman in the district hospital, maternity ward

Спостерігати протягом доби Watch during the day

171 / 200
Хворого госпіталізовано з підозрою на гостру кишкову непрохідність. Який з методів дослідження буде найбільш інформативним для підтвердження діагнозу? The patient is hospitalized with suspicion of acute intestinal obstruction. Which of the research methods will be the most informative to confirm the diagnosis?

Лапароцентез Laparocentesis

Лапароскопія Laparoscopy

ФГДС FGDS

УЗД Ultrasound

Оглядова рентгенографія органів черевної порожнини Overview x-ray of abdominal organs

172 / 200
Хвора 38-ми років скаржиться на 'припливи' і відчуття жару, які повторюються до 5 разів на добу, головний біль у потиличній ділянці з підвищенням артеріального тиску, серцебиття, запаморочення, швидку втомлюваність, дратівливість, погіршення пам’яті. 6 місяців тому проведена операція в об’ємі екстирпації матки з придатками. Який найбільш імовірний діагноз? A 38-year-old patient complains of 'flushes' and a feeling of heat, which are repeated up to 5 times a day, headache in the occipital region with increased blood pressure, palpitations , dizziness, rapid fatigue, irritability, memory impairment. 6 months ago, an operation was performed to extirpate the uterus with appendages. What is the most likely diagnosis?

Післякастраційний синдром Post-castration syndrome

Ранній патологічний клімакс Early pathological menopause

Вторинна психогенна аменорея Secondary psychogenic amenorrhea

Передменструальний синдром Premenstrual syndrome

Фізіологічна пременопауза Physiological premenopause

173 / 200
3 дні назад у хлопчика з-під нігтьової пластинки видалено стороннє тіло. Через 2 дні з’явився різкий пульсуючий біль у кінці нігтьової фаланги, особливо при натисканні, гіперемія нігтьового валика, піднялась температура тіла до 37,5оC; колір нігтьової пластинки не змінився. Який найбільш імовірний діагноз? 3 days ago, the boy had a foreign body removed from under the nail plate. 2 days later, a sharp throbbing pain appeared at the end of the nail phalanx, especially when pressed, hyperemia of the nail plate, the body temperature rose to 37.5°C; the color of the nail plate did not change. What is the most likely diagnosis?

Пароніхія Paronychia

Еризипелоїд Erysipeloid

Абсцес Abscess

Піднігтьовий панарицій Subungual panaritium

Бешиха Beshikha

174 / 200
Під час огляду пацієнта 32-х років, з надлишковою вагою, шкіра міжпальцевих ділянок обох ступнів еритематозна, мацерована, дрі6но лущиться, у складках між V, IV та III пальцями є поодинокі тріщини, що мокнуть. Турбує незначний свербіж. Який найбільш імовірний діагноз? During the examination of a 32-year-old patient, with excess weight, the skin of the interdigital areas of both feet is erythematous, macerated, poorly peeling, in the folds between V, IV and III there are isolated cracks that wet the fingers. There is slight itching. What is the most likely diagnosis?

Рубромікоз Rubromycosis

Епідермомікоз Epidermomycosis

Алергічний дерматит Allergic dermatitis

Мікробна екзема Microbial Eczema

Стрептодермія Streptoderma

175 / 200
Внаслідок вибуху цистерни з бензолом на хімічному заводі наявні загиблі та поранені у великій кількості (понад 50 осіб) з опіками, механічними травмами та отруєннями. Вкажіть основні елементи, які передбачає лікувально-евакуаційне забезпечення населення в цій ситуації: As a result of the explosion of a tank with benzene at a chemical plant, there are a large number of dead and injured (over 50 people) with burns, mechanical injuries and poisoning. Specify the main elements that provides medical and evacuation support for the population in this situation:

Сортування, відновлення, рятування Sort, Restore, Rescue

Сортування, надання медичної допомоги, евакуація Sort, medical assistance, evacuation

Надання медичної допомоги, евакуація, ізоляція Medical assistance, evacuation, isolation

Сортування, евакуація, лікування Triage, evacuation, treatment

Ьоляція, рятування, відновлення Olation, Rescue, Recovery

176 / 200
Жінка 40-ка років з 15-ти років страждає на епілепсію з генералізованими епінападами, які виникають 2-3 рази на місяць у нічний час та супроводжуються мимовільним сечовиділенням та дефекацією. Після психотравми (смерть батька) напади почастішали, виникають з періодичністю 2-3 хвилини, між нападами до свідомості не повертається, реакція зіниць на світло відсутня. Який найбільш імовірний діагноз? A 40-year-old woman has been suffering from epilepsy since the age of 15 with generalized epileptic seizures that occur 2-3 times a month at night and are accompanied by involuntary urination and defecation. After psychotrauma (father's death), the attacks became more frequent, occur with a frequency of 2-3 minutes, he does not return to consciousness between attacks, the reaction of the pupils to light is absent. What is the most likely diagnosis?

!стеричний напад !steric attack

Абсанс Absence

Збільшення кількості великих епіна-падів Increasing the number of large epinapads

Епілептичний статус Status epilepticus

Епілептичний психоз Epileptic psychosis

177 / 200
Хворий 52-х років постраждав, коли в під’їзді будинку спрацював невідомий вибуховий пристрій. В ділянці лівої поверхні шиї, в проекції m. Strenoqleudomastoideus спостерігаються три осколкових поранення діаметром 2х3 мм, з яких постійно поступає темна кров. Спостерігається різко виражений набряк лівої половини шиї, тут же відмічається гематома, що постійно збільшується, непульсуюча. Визначте обсяг першої допомоги: A 52-year-old patient was injured when an unknown explosive device went off in the entrance of the house. There are three shrapnel wounds in the area of the left surface of the neck, in the projection of m. Strenoqleudomastoideus with a diameter of 2x3 mm, from which dark blood is constantly flowing. A sharply expressed swelling of the left half of the neck is observed, a constantly increasing, non-pulsating hematoma is immediately noted. Determine the scope of first aid:

Накладання оклюзійної пов’язки Applying an occlusive dressing

Застосування протромбінової губки Application of prothrombin sponge

Накладання джгута Applying harness

Накладання стискаючої пов’язки Applying compression bandage

!ммобілізація шийного відділу хребта комірцем Шанца !mmobilization of the cervical spine with a Shantz collar

178 / 200
Хворий 39-ти років скаржиться на задишку при фізичному навантаженні, загальну слабкість, пітливість. Об’єктивно: бочкоподібна грудна клітка, ЧД- 20/хв., Ps- 84/хв., ритмічний. АТ-130/90 мм рт.ст. Перкуторно: коробковий відтінок легеневого звуку, дихання послаблене, тони серця приглушені, ритмічні. Периферійні набряки відсутні. Яке порушення функції зовнішнього дихання буде у хворого? A 39-year-old patient complains of shortness of breath during physical exertion, general weakness, sweating. Objectively: barrel-shaped chest, HR-20/min., Ps - 84/min., rhythmic. BP-130/90 mm Hg. Percussion: box tone of lung sound, breathing is weakened, heart sounds are muffled, rhythmic. Peripheral edema is absent. What disturbance of the function of external breathing will the patient have?

Нормальні показники Normal indicators

Зниження дихальних об’ємів Decreased respiratory volumes

Підвищення швидких параметрів вдиху за секунду Increasing fast breath settings per second

Нападоподібне погіршення параметрів 'потік-об’єм' Strike-like degradation of parameters 'flow-volume'

Зниження пікової швидкості вдиху при збереженні параметрів видиху Decreasing peak inspiratory speed while maintaining exhalation parameters

179 / 200
Хворий 39-ти років скаржиться на підвищення температури тіла до 41oC, головний біль, слабкість. Захворів гостро, на 5-й день захворювання з’явився розеольозно-петехіальний висип на бічній поверхні грудної клітки, спини. РЗК з рикетсіями Провачека 1:640, lgM - 89%. Який найбільш імовірний діагноз? A 39-year-old patient complains of an increase in body temperature up to 41oC, headache, weakness. He became acutely ill, on the 5th day of the disease roseolosis-petechial a rash on the lateral surface of the chest, back. RZK with rickettsia Provacek 1:640, lgM - 89%. What is the most likely diagnosis?

Грип Flu

Висипний тиф Typhoid

Черевний тиф Typhoid

Ентеровірусна інфекція Enterovirus infection

Хвороба Брила Bril's disease

180 / 200
Пацієнтка 55-ти років, у якої менструації припинились 5 років тому, скаржиться на відчуття сухості в піхві, часте та болісне сечовипускання. Лікар-гінеколог при обстеженні виявив ознаки атрофічного кольпіту. У сечі: без особливостей. Які засоби місцевої дії дадуть належний терапевтичний ефект? A 55-year-old female patient, whose menstruation stopped 5 years ago, complains of a feeling of dryness in the vagina, frequent and painful urination. During the examination, the gynecologist found signs atrophic colpitis. In the urine: no specifics. What means of local action will give the proper therapeutic effect?

Вагінальний крем 'Далацин' Vaginal cream 'Dalacin'

Вагінальний крем 'Меротин-Комбі' Vaginal cream 'Merotin-Combi'

Вагінальні таблетки 'Тержинан' Vaginal tablets 'Terzhinan'

Вагінальні свічки 'Овестін' Vaginal suppositories 'Ovestin'

Вагінальний гель 'Метронідазол' Vaginal gel 'Metronidazole'

181 / 200
Хворий 63-х років з постійною формою фібриляції передсердь, скаржиться на помірну задишку. Об’єктивно: периферичні набряки відсутні, дихання везикулярне, ЧСС- 72/хв., АТ-140/90 мм рт.ст. Який комплекс медикаментів є найбільш доцільним у вторинній профілактиці серцевої недостатності? A 63-year-old patient with persistent atrial fibrillation complains of moderate shortness of breath. Objectively: no peripheral edema, vesicular breathing, heart rate - 72/min. , BP-140/90 mm Hg. What complex of medications is the most appropriate in the secondary prevention of heart failure?

в-адреноблокатори, серцеві глікозиди v-adrenoblockers, cardiac glycosides

в-адреноблокатори, іАПФ v-adrenoblockers, iACE

Серцеві глікозиди, іАПФ Cardiac glycosides, iACE

Діуретики, в-адреноблокатори Diuretics, β-adrenergic blockers

Серцеві глікозиди, діуретики Cardiac glycosides, diuretics

182 / 200
У жінки 40-ка років 5 років тому проведена двостороння аднексектомія. Скаржиться на слабкість, втомлюваність, відсутність менструації, біль у попереку, зниження пам’яті, не пам’ятає чинні події. При обстеженні наявність ожиріння, остеопорозу та гіперхолестеринемії. Який синдром спостерігається у пацієнтки? A 40-year-old woman underwent bilateral adnexectomy 5 years ago. She complains of weakness, fatigue, lack of menstruation, lower back pain, memory loss, 'reports current events. During the examination, the presence of obesity, osteoporosis, and hypercholesterolemia. What syndrome is observed in the patient?

Адіпозо-генітальна дистрофія Adipose-genital dystrophy

Антифосфоліпідний синдром Antiphospholipid syndrome

Клімактеричний синдром Climacteric syndrome

Післякастраційний синдром Post-castration syndrome

Адреногенітальний синдром Adrenogenital syndrome

183 / 200
Хлопчик 8-ми років впродовж 2-х років страждає на появу плямистих висипів та свербіж, які виникають після вживання цитрусових. З анамнезу: вперше висипка виникла у 6 місяців після введення до раціону харчування соків. Батько страждає на бронхіальну астму, мати - на алергічний риніт. Який діагноз можна припустити? An 8-year-old boy has been suffering from spotty rashes and itching after eating citrus fruits for 2 years. From the anamnesis: the rash first appeared at 6 months after the introduction of juices into the diet. The father suffers from bronchial asthma, the mother - from allergic rhinitis. What diagnosis can be assumed?

Розовий лишай Pink lichen

Кропив’янка Hives

Набряк Квінке Quincke edema

Псоріаз Psoriasis

Атопічний дерматит Atopic dermatitis

184 / 200
На 10-ту добу післяпологового періоду породілля скаржиться на болі в молочних залозах. Температура тіла -38,2oC, Ps- 96/хв. В молочних залозах значний і рівномірний набряк, болі під час пальпації. При натисненні із сосків виділяються крапельки молока. Яка тактика по відношенню до цієї хворої? On the 10th day of the postpartum period, a woman in labor complains of pain in the mammary glands. Body temperature -38.2oC, Ps- 96/min. In the mammary glands, significant and uniform swelling, pain during palpation. When pressed, drops of milk are released from the nipples. What are the tactics in relation to this patient?

Припинення лактації Stop lactation

Призначити антибіотикотерапію та сечогінне Prescribe antibiotic therapy and diuretics

Компрес на молочні залози Breast compress

Тимчасове обмеження обсягу рідини в раціоні Temporary restriction of the amount of liquid in the diet

Спорожнити груди шляхом зціджування або за допомогою молоковідсмо-ктувача Empty the breast by pumping or using a breast pump

185 / 200
8-ми років із скаргами на підвищення температури тіла до 39,8o 185. C, млявість, помірний головний біль, блювання. При огляді виявлені менінгеальні симптоми. Проведено люмбальну пункцію. Отримано рідину під підвищеним тиском, прозору, цитоз 450 клітин в 1 мкл (переважно лімфоцити - 90%), вміст глюкози 2,6 ммоль/л. Яким збудником може бути викликано захворювання у дитини? 8-year-old with complaints of an increase in body temperature up to 39.8o 185. C, lethargy, moderate headache, vomiting. During the examination, meningeal symptoms were revealed. Conducted lumbar puncture. Fluid under increased pressure was obtained, transparent, cytosis of 450 cells in 1 μl (mostly lymphocytes - 90%), glucose content 2.6 mmol/l. What pathogen can cause the disease in a child?

Менінгокок Meningococcus

Туберкульозна паличка Tuberculosis bacillus

Ентеровірус Enterovirus

Стафілокок Staphylococcus

Пневмокок Pneumococcus

186 / 200
Хвора 25-ти років під час самообстеження виявила пухлину у верхньому зовнішньому квадранті правої молочної залози. При пальпації - безболісне, тверде, рухоме утворення молочної залози діаметром 2 см, периферичні лімфатичні вузли не змінені. При ультразвуковому дослідженні молочних залоз: у верхньому зовнішньому квадранті правої молочної залози об’ємне утворення підвищеної ехогенності, розміром 21х18 мм. Який найбільш імовірний діагноз? During a self-examination, a 25-year-old patient discovered a tumor in the upper outer quadrant of the right mammary gland. On palpation - a painless, hard, mobile mass of the mammary gland with a diameter of 2 cm, peripheral lymph nodes are not changed. During the ultrasound examination of the mammary glands: in the upper outer quadrant of the right mammary gland there is a volume formation of increased echogenicity, the size of 21x18 mm. What is the most likely diagnosis?

Мастит Mastitis

Дифузна мастопатія Diffuse mastopathy

Кіста молочної залози Breast gland cyst

Рак молочної залози Breast cancer

Фіброаденома Fibroadenoma

187 / 200
Хвора 23-х років скаржиться на наявність пухлини у нижньому зовнішньому квадранті лівої молочної залози протягом 1-го року, що перед менструацією стає болісним і збільшується в розмірах. При пальпації: рухоме утворення, наповнене рідиною, до 3 см, із чіткими контурами, периферичні лімфатичні вузли не змінені. При ультразвуковому дослідженні молочних залоз: у нижньому зовнішньому квадранті лівої молочної залози об’ємне утворення зниженої ехогенності, розміром 31х29 мм. Який попередній діагноз? A 23-year-old patient complains of the presence of a tumor in the lower outer quadrant of the left mammary gland during the 1st year, which becomes painful and increases in size before menstruation. palpation: a mobile mass filled with liquid, up to 3 cm, with clear contours, peripheral lymph nodes are not changed. During ultrasound examination of the mammary glands: in the lower outer quadrant of the left mammary gland, a volumetric mass of reduced echogenicity, measuring 31x29 mm. What is the previous diagnosis?

Кіста молочної залози Breast gland cyst

Фіброзна мастопатія Fibrous mastopathy

Рак молочної залози Breast cancer

Масталгія Mastalgia

Фіброаденома Fibroadenoma

188 / 200
Хворий 50-ти років протягом 15-ти років працював на хімічному заводі. В роботі використовував розчинник - ксилол. Був госпіталізований з підозрою на хронічну інтоксикацію. Встановлено анемічний синдром. Який захід вторинної профілактики анемії буде першочерговим ? A 50-year-old patient worked at a chemical plant for 15 years. He used the solvent - xylene at work. He was hospitalized with suspicion of chronic intoxication. Anemic syndrome was diagnosed What measure of secondary prevention of anemia will be the first priority?

Зміна місця роботи Change of place of work

Включення в їжу м’ясних продуктів Inclusion of meat products in food

Включення в дієту морепродуктів Inclusion of seafood in the diet

Призначення глюкокортикоїдів Prescription of glucocorticoids

Застосування залізовмісних препаратів Use of iron-containing preparations

189 / 200
Через 2 тижні після пологів у породіллі з’явилися болі в молочній залозі, що наростали впродовж 3-х днів. Об’єктивно: температура тіла 39oC, озноб, слабкість, гіперемія шкіри, збільшення, болючість та деформація молочної залози. При пальпації інфільтрату визначається ділянка розм’якшення та флюктуації. Який найбільш імовірний діагноз? 2 weeks after giving birth, the mother developed pain in the mammary gland, which increased over the course of 3 days. Objectively: body temperature 39oC, chills, weakness, hyperemia of the skin, enlargement, tenderness and deformation of the mammary gland. During palpation of the infiltrate, an area of softening and fluctuation is determined. What is the most likely diagnosis?

Мастопатія Mastopathy

Інфільтративно-гнійний мастит Infiltrative-purulent mastitis

Серозний мастит Serous mastitis

Лактостаз Lactostasis

Флегмонозний мастит Phlegmonous mastitis

190 / 200
Хворий 38-ми років скаржиться на утрудненне ковтання твердої їжі, яке з’явилось біля місяця тому, а в останній час і напіврідкої. Відмічає також слабкість, погіршення загального стану, зниження апетиту, втрату маси тіла, підвищення температури. Об’єктивно: шкіра суха, бліда, язик обкладений, інших відхилень не виявлено. У крові: гіпохромна анемія, підвищення ШОЕ. Який попередній діагноз? A 38-year-old patient complains of difficulty swallowing solid food, which appeared about a month ago, and more recently of semi-liquid food. He also notes weakness, deterioration of the general condition, decreased appetite, loss of body weight, increased temperature. Objectively: the skin is dry, pale, the tongue is coated, no other abnormalities were detected. In the blood: hypochromic anemia, increased ESR. What is the previous diagnosis?

Рак шлунку Stomach cancer

Езофагіт Esophagitis

Склеродермія Scleroderma

Рак стравоходу Esophageal cancer

Гастроезофагеальна рефлюксна хвороба Gastroesophageal reflux disease

191 / 200
Хворий 70 -ти років звернувся до лікаря зі скаргами на неритмічність серцевої діяльності, задишку. Об’єктивно: АТ- 150/90 мм рт.ст., екстрасистолічна аритмія (10-12 екстрасистол за хвилину), систолічна дисфункція лівого шлуночка (фракція викиду 42%). Який із антиаритмічних препаратів в якості початкової терапії слід призначити в даному випадку? A 70-year-old patient consulted a doctor with complaints of cardiac arrhythmia, shortness of breath. Objectively: blood pressure - 150/90 mmHg, extrasystolic arrhythmia (10-12 extrasystoles per minute), systolic dysfunction of the left ventricle (ejection fraction 42%). Which of the antiarrhythmic drugs should be prescribed as initial therapy in this case?

Дигоксин Digoxin

Енкаінід Encainide

Аміодарон Amiodarone

Флекаінід Flecainide

Морацизин Moracizin

192 / 200
Хворий 45-ти років висловлює скарги на інтенсивний різкий біль у правому боці, що іррадіює у праве стегно і промежину. Відзначає часті позиви до сечовипускання, сечу кольору 'м’ясних помиїв'. Подібний стан описує вперше. Позитивний симптом Пастернацького справа. Який найбільш імовірний діагноз? A 45-year-old patient complains of intense, sharp pain in the right side, radiating to the right thigh and perineum. He notes frequent urges to urinate, urine color 'clear scum'. This is the first time he describes a similar condition. A positive symptom of Pasternaksky's case. What is the most likely diagnosis?

Сечокам’яна хвороба Urolithiasis

Гострий холецистит. Ниркова колька Acute cholecystitis. Renal colic

Гострий панкреатит Acute pancreatitis

Гострий апендицит Acute appendicitis

Гострий пієлонефрит Acute pyelonephritis

193 / 200
В ході розслідування випадку масового отруєння у автомеханіків, які проводили випробування дизельних двигунів у боксі майстерні, де була зіпсована витяжна вентиляція, наприкінці робочого дня з’явились симптоми: головний біль, нудота, блювання, шум у вухах, лабільність пульсу. Об’єктивно: шкіра та слизові оболонки вишнево-червоного кольору. Який токсичний чинник став причиною масового отруєння автомеханіків? During the investigation of the case of mass poisoning, auto mechanics who were testing diesel engines in the workshop box, where the exhaust ventilation was damaged, at the end of the working day showed symptoms: the main pain, nausea, vomiting, tinnitus, pulse lability. Objectively: cherry-red skin and mucous membranes. What toxic factor caused the mass poisoning of auto mechanics?

Сірковуглець Carbon disulfide

Оксид вуглецю Carbon monoxide

Оксид азоту Nitrogen oxide

Діоксид сірки Sulfur dioxide

Діоксид вуглецю Carbon dioxide

194 / 200
Хворого 56-ти років доставлено бригадою швидкої допомоги з носовою кровотечею. Хворіє на гіпертонічну хворобу. На момент огляду: АТ- 200/130 мм рт.ст., введено гіпотензивні засоби. Об’єктивно: з носової порожнини виділяється кров, переважно затікаючи в глотку. Одноразове блювання згустками крові. Яку допомогу слід надати хворому? A 56-year-old patient was delivered by an ambulance with a nosebleed. He suffers from hypertension. At the time of examination: blood pressure - 200/130 mm Hg, antihypertensive agents were introduced. Objectively: blood is coming out of the nasal cavity, mainly flowing into the pharynx. One-time vomiting of blood clots. What help should be given to the patient?

Введення гемостатичних та гіпотензивних засобiв Introduction of hemostatic and hypotensive agents

Перев’язка зовнішньої сонної артерії Ligation of external carotid artery

Електрокоагуляція кровоточивої судини Electrocoagulation of a bleeding vessel

Передня тампонада носа Anterior nasal tamponade

Задня тампонада носа Posterior nasal tamponade

195 / 200
Хвора з ожирінням II ступеня (ІМТ 36 кг/м2) звернулась до ендокринолога з метою схуднути. Який вид дієтотерапії слід запропонувати хворій? A patient with II degree obesity (BMI 36 kg/m2) consulted an endocrinologist with the aim of losing weight. What type of diet therapy should be offered to the patient?

Дієта з обмеженням вуглеводів Carbohydrate-restricted diet

Дієта з обмеженням жирів Low-fat diet

Голодування протягом 2-х тижнів Fasting for 2 weeks

Фізіологічна субкалорійна дієтотерапія Physiological subcaloric diet therapy

Дієта з обмеженням білків Protein restricted diet

196 / 200
Пацієнтка 20-ти років звернулась до лікаря жіночої консультації зі скаргами на відсутність менструацій протягом 7-ми місяців. З анамнезу: в ранньому віці хворіла на дитячі інфекції і ангіни, менархе з 13-ти років, місячні регулярні, менструальний цикл 28 днів, менструація триває 5-6 днів, безболісна. 7 місяців тому перенесла стрес. При гінекологічному огляді змін з боку матки і додатків не виявлено. Який найбільш імовірний діагноз? A 20-year-old patient turned to the doctor of the women's consultation with complaints about the absence of menstruation for 7 months. From the anamnesis: she suffered from childhood infections and sore throats at an early age , menarche since the age of 13, periods are regular, the menstrual cycle is 28 days, menstruation lasts 5-6 days, painless. 7 months ago, she experienced stress. During a gynecological examination, no changes were detected in the uterus and appendages. What is the most likely diagnosis?

Вторинна аменорея Secondary amenorrhea

Несправжня аменорея False amenorrhea

Альгодисменорея Algodysmenorrhea

Олігоменорея Oligomenorrhea

Первинна аменорея Primary amenorrhea

197 / 200
Хлопчик 4-х років прокинувся вночі внаслідок нападу кашлю, задишки. В анамнезі - атопічний дерматит. Стан порушений: дихальна недостатність II ступеня. Дитина бліда, перелякана, грудна клітка здута, вислуховуються дистантні хрипи. Над легенями - тимпаніт, видих значно подовжений, розсіяні сухі та вологі хрипи з обох боків. Чим обумовлені виявлені зміни? A 4-year-old boy woke up at night due to an attack of coughing, shortness of breath. He has a history of atopic dermatitis. The condition is disturbed: respiratory insufficiency of the II degree. The child is pale, frightened, chest the cage is inflated, distant wheezes are heard. Tympanitis over the lungs, exhalation is significantly prolonged, scattered dry and wet wheezes on both sides. What are the reasons for the detected changes?

Бронхообструктивний синдром Broncho-obstructive syndrome

Стенозуючий ларинготрахеїт Stenotic laryngotracheitis

Аспірація стороннього тіла Foreign body aspiration

Стридор Stridor

Пневмонія Pneumonia

198 / 200
Хворий 30-х років поступив до протитуберкульозного диспансеру у зв’язку з виявленими на флюорографії змінами: у S1 правої легені відмічається тінь діаметром до 1 см, слабкої інтенсивності з нечіткими контурами. На томограмі у центрі тіні визначається деструкція. У харкотинні виявлено МБТ. Хворому встановлено діагноз вогнищевого туберкульозу. Яким фазам туберкульозного процесу відповідають виявлені зміни? A patient in his 30s was admitted to the anti-tuberculosis dispensary in connection with changes detected on fluorography: in S1 of the right lung, a shadow with a diameter of up to 1 cm, of weak intensity with with indistinct contours. Destruction is determined in the center of the shadow on the tomogram. MBT was detected in the sputum. The patient was diagnosed with focal tuberculosis. What phases of the tuberculosis process do the detected changes correspond to?

Інфільтрації та розпаду Infiltration and Decay

Розсмоктування і рубцювання Resorption and scarring

Розпаду і обсіменіння Decomposition and Insemination

Інфільтрації та обсіменіння Infiltrations and Inseminations

Ущільнення і розсмоктування Consolidation and dissolution

199 / 200
У постраждалого з ножовим пораненням грудної клітки справа визначається ціаноз, АТ70/40 мм рт.ст., Ps-120/хв., ЧД- 34/хв., відставання правої половини грудної клітки при диханні. Першочерговим заходом надання допомоги буде: Cyanosis is determined in the victim with a knife wound to the chest on the right side, blood pressure 70/40 mm Hg, Ps-120/min., BH- 34/min. , lagging of the right half of the chest during breathing. The first aid measure will be:

Міжреберна блокада Intercostal blockade

Пункція правої плевральної порожнини Puncture of the right pleural cavity

Введення препаратів крові Introduction of blood products

Введення інотропних фармакологічних препаратів Introduction of inotropic pharmacological drugs

Негайне переведення хворого на ШВЛ Immediate transfer of the patient to ventilator

200 / 200
У структурі населення регіону питома вага осіб віком від 0 до 14 років - 25%, питома вага осіб від 50 років і старше - 30%. Яке поняття найбільш точно характеризує цю демографічну ситуацію? In the structure of the population of the region, the specific weight of people aged 0 to 14 years is 25%, the specific weight of people 50 years and older is 30%. Which concept is the most accurate characterizes this demographic situation?

Стаціонарний тип вікової структури населення Stationary type of population age structure

Когортний тип населення Cohort population type

Регресивний тип вікової структури населення Regressive type of population age structure

Прогресивний тип вікової структури населення Progressive type of population age structure

Імміграція населення Population immigration




1
2
3
4
5
6
7
8
9
10
11
12
13
14
15
16
17
18
19
20
21
22
23
24
25
26
27
28
29
30
31
32
33
34
35
36
37
38
39
40
41
42
43
44
45
46
47
48
49
50
51
52
53
54
55
56
57
58
59
60
61
62
63
64
65
66
67
68
69
70
71
72
73
74
75
76
77
78
79
80
81
82
83
84
85
86
87
88
89
90
91
92
93
94
95
96
97
98
99
100
101
102
103
104
105
106
107
108
109
110
111
112
113
114
115
116
117
118
119
120
121
122
123
124
125
126
127
128
129
130
131
132
133
134
135
136
137
138
139
140
141
142
143
144
145
146
147
148
149
150
151
152
153
154
155
156
157
158
159
160
161
162
163
164
165
166
167
168
169
170
171
172
173
174
175
176
177
178
179
180
181
182
183
184
185
186
187
188
189
190
191
192
193
194
195
196
197
198
199
200

Завершити тест